Nothing Special   »   [go: up one dir, main page]

HS 330 1-1

Download as doc, pdf, or txt
Download as doc, pdf, or txt
You are on page 1of 231

The American College HS 330

Fundamentals of Estate Planning Test


Version 1.1
QUESTION NO: 1

Many trust instruments provide for the removal of the original trustee. Valid reasons for removing the
original trustee include which of the following?

1. A shift in trust situs is desirable because of changes in law.

2. The beneficiary has moved his or her residence to a distant state.

A. Neither 1 nor 2

B. 1 only

C. Both 1 and 2

D. 2 only

Answer: C

QUESTION NO: 2

In which of the following situations will the grantor be taxed on income from trust property.

1. The grantor of a trust gives one of the trust beneficiaries the right to add or delete beneficiaries.

2. An adverse party to the grantor holds the power to determine the timing of trust distributions to
the beneficiaries.
A. 1 only

B. Both 1 and 2

C. 2 only

D. Neither 1 nor 2

Answer: D

QUESTION NO: 3

The following are facts concerning a decedent's estate:

 Taxable estate $1,800,000


 Pre-1977 taxable gifts 100,000
 Post-1976 adjusted taxable gifts 150,000
 Post-1976 gifts made to a qualified charity 200,000

The tentative tax base of this estate is

A. $1,800,000

B. $1,950,000

C. $1,650,000

D. $2,150,000

Answer: B

QUESTION NO: 4
Items that are deductions from a decedent's gross estate in determining his adjusted gross estate
include which of the following?

1. Foreign death taxes

2. State death taxes

A. Neither 1 nor 2

B. Both 1 and 2

C. 1 only

D. 2 only

Answer: A

QUESTION NO: 5

All the following trust provisions avoid causing the inclusion of an irrevocable life insurance trust in an
insured's gross estate EXCEPT

A. a trustee's power to pay estate expenses

B. a trust beneficiary's power to withdraw contributions to the trust

C. a trustee's power to loan assets to the estate at the trustee's discretion

D. a trustee's power to purchase assets from the estate at the trustee's discretion

Answer: A
QUESTION NO: 6

Which of the following terms applies to the blending together of separate and community properties of
spouses in community-property states?

A. Inception

B. Transmutation

C. Commingling

D. Proration

Answer: C

QUESTION NO: 7

All the following statements concerning wills are correct EXCEPT:

A. Once a person is named as an executor in a will, he or she is required to serve.

B. A codicil is a valid modification of a will.

C. A testator may lose the capacity to revoke a will prior to death.

D. In most states a surviving spouse can elect against a will that completely disinherits him or her.

Answer: A

QUESTION NO: 8
If a grantor establishes an irrevocable trust, the income of the trust will be taxed to the grantor if it is
used to pay premiums for life insurance on the life of

A. a child of the grantor

B. the father of the grantor

C. the spouse of the grantor

D. a grandchild of the grantor

Answer: C

QUESTION NO: 9

Which of the following statements concerning ownership of property in the form of a joint tenancy with
right of survivorship is correct?

A. Qualified joint tenancies cannot be severed without mutual consent.

B. Upon the death of a joint tenant, the surviving joint tenant receives the decedent's interest by
operation of law.

C. Nonqualified joint tenants may have unequal interests in the property.

D. This form of ownership is limited to real property.

Answer: B

QUESTION NO: 10

Which of the following statements concerning executors is correct?


A. An executor can be given broad and discretionary powers with respect to the management of the
estate.

B. All executors must post bonds.

C. The executor must be a licensed attorney in the decedent's state of domicile.

D. The executor named in the will of a deceased person is automatically authorized to act.

Answer: A

QUESTION NO: 11

Which of the following statements concerning the generation-skipping transfer tax (GSTT) is (are)
correct?

1. An annual exclusion against GSTT will shelter gifts by a grandparent to a trust benefitting
multiple grandchildren.

2. Tuition payments made directly by a grandparent to a university for a grandchild's education are
exempt from GSTT.

A. Neither 1 nor 2

B. 1 only

C. Both 1 and 2

D. 2 only

Answer: D
QUESTION NO: 12

All the following statements concerning real property ownership by married couples as joint tenants
with right of survivorship are correct EXCEPT:

A. All benefits of ownership remain available to the surviving spouse without interruption during the
administration of the deceased spouse's estate.

B. In common-law states the total value of the property receives a stepped-up tax basis in the estate of
the first spouse to die.

C. Jointly held property between spouses does not pass through the probate estate of the first spouse to
die.

D. The deceased spouse's interest in the property qualifies for the marital deduction since it passes
outright to the surviving spouse.

Answer: B

QUESTION NO: 13

Many trust instruments provide for the removal of the original trustee. All the following are valid
reasons for removal of a trustee EXCEPT:

A. A shift in trust situs is desirable because of changes in law.

B. The beneficiary is not able to get along with the present corporate trustee.

C. The beneficiary has moved his or her residence to a distant state.

D. The beneficiary has been successful with investments and wants to manage the trust assets.

Answer: D
QUESTION NO: 14

A father bought stock for $100,000 and gave it to his son when it was worth $300,000. The father paid
no gift tax on the transfer. When the son sold the property 2 years after the gift, his income tax basis
was

A. $100,000

B. 0

C. $300,000

D. $200,000

Answer: A

QUESTION NO: 15

A father deeded a house as a gift to his daughter in 1990 but retained the right to live in it until his
death. He died this year while still living in the house. The following are relevant facts: The father bought
the property in 1980 for $130,000. The fair market value of the property when the gift was made in
1990 was $150,000. The father filed a timely gift tax return but paid no gift tax because of the
applicable credit amount. The fair market value of the property at the father's death was $220,000. The
daughter sold the property 3 months after her father's death for $220,000. She had a gain of

A. $120,000

B. 0

C. $140,000

D. $220,000

Answer: B
QUESTION NO: 16

A father wants to accumulate funds for his 12-year-old son's college education. On the advice of his
attorney, the father establishes an IRC Section 2503(c) trust and funds it with annual gifts. All the
following statements concerning this arrangement are correct EXCEPT:

A. In the event of the son's death prior to age 21, trust assets must either be payable to the son's estate
or be subject to a general power of appointment held by the son.

B. The father's annual gift tax exclusion must be reduced by any amount used to pay college tuition
costs.

C. The trust must be irrevocable.

D. Any accumulated income and all trust principal must be available for distribution to the son when he
attains age 21.

Answer: B

QUESTION NO: 17

Which of the following statements concerning the estate tax marital deduction is correct?

A. The marital deduction available to a decedent in a common-law state is limited to a maximum of $1


million.

B. The marital deduction available to a decedent in a common-law state is equal to the net amount of
qualifying property passing to the surviving spouse.

C. The marital deduction available to a decedent in a community-property state is equal to the total
amount of community property.

D. The marital deduction available to a decedent in a common-law state is equal to one half the adjusted
gross estate.
Answer: B

QUESTION NO: 18

All the following statements concerning real property ownership by married couples as joint tenants
with right of survivorship are correct EXCEPT:

A. Jointly held property between spouses does not pass through the probate estate of the first spouse to
die.

B. In common-law states the total value of the property receives a stepped-up tax basis in the estate of
the first spouse to die.

C. All benefits of ownership remain available to the surviving spouse without interruption during the
administration of the deceased spouse's estate.

D. The deceased spouse's interest in the property qualifies for the marital deduction since it passes
outright to the surviving spouse.

Answer: B

QUESTION NO: 19

A wife makes outright gifts of $40,000 to her son this year, and her husband agrees to split the gifts with
her. Which of the following correctly states the amount of the taxable gifts?

A. Wife $9,000, husband $9,000

B. Wife 0, husband $18,000

C. Wife $18,000, husband 0

D. Wife $19,000, husband $19,000


Answer: A

QUESTION NO: 20

Which of the following transactions is a taxable gift for federal gift tax purposes?

A. A grandmother purchased a U.S. savings bond that is registered as payable to her and her two
grandchildren and the bond has not yet been surrendered for cash.

B. A father made a deposit of $100,000 into a bank account titled jointly with his son, and the son has
not yet made any withdrawals.

C. A mother purchased listed common stocks titling them in joint names with her daughter and the
stocks have not yet been sold.

D. A man deeded real estate to his sister but did not record the deed nor did he deliver the deed to his
sister.

Answer: C

QUESTION NO: 21

A father plans to create a trust for the benefit of his 22-year-old son and wishes to take advantage of the
gift tax annual exclusion. He has named a bank as trustee. Which of the following trust provisions would
cause the gifts to be ineligible to qualify for the gift tax annual exclusion?

1. The trust income is to be paid to the son or accumulated at the discretion of the trustee.

2. The income is to be accumulated until the son reaches age 32 when all accumulated income and
principal are to be distributed to him.
A. 1 only

B. Neither 1 nor 2

C. Both 1 and 2

D. 2 only

Answer: C

QUESTION NO: 22

Which of the following statements concerning the inclusion and valuation of all or part of a commercial
annuity in the estate of an annuitant is (are) correct?

1. A life annuity with a period certain is includible to the extent of the present value of any remaining
guaranteed payments.

2. If the executor elects the alternate valuation date, an annuity is includible at its replacement cost 6
months after death.

A. Neither 1 nor 2

B. Both 1 and 2

C. 2 only

D. 1 only

Answer: D

QUESTION NO: 23
Generally all the following statements concerning life insurance arrangements for partnership buy-sell
agreements are correct EXCEPT:

A. With an entity-purchase agreement the formula used for determining the number of life insurance
policies needed is N (number of partners) + 1.

B. With an entity-purchase agreement the partnership strives to maintain face amounts of coverage
equal to its obligations under the agreement.

C. With a cross-purchase agreement each partner purchases life insurance on the life of each of the
other partners.

D. With a cross-purchase agreement the individual partners are the applicants, owners, beneficiaries
and premium-payers of the policies.

Answer: A

QUESTION NO: 24

Which of the following statements concerning a testamentary trust is correct?

A. It becomes effective only at the death of the grantor.

B. It saves federal and state death taxes at the death of the grantor.

C. The trust terms must be included in the will.

D. The assets in the trust are free of probate costs.

Answer: A

QUESTION NO: 25
Ignoring the annual per-donee exclusion, which of the following transfers is a gift for federal gift tax
purposes?

A. A creditor cancels the promissory note of a recently unemployed friend as a charitable gesture.

B. A father promises to buy his daughter a condominium when she finishes college.

C. A grandmother pays her grandson's $30,000 tuition at an Ivy League university.

D. An individual gratuitously performs valuable services for the benefit of a close friend.

Answer: A

QUESTION NO: 26

Which of the following statements concerning ownership of property in the form of a joint tenancy with
right of survivorship is correct?

A. This form of ownership is limited to real property.

B. Nonqualified joint tenants may have unequal interests in the property.

C. Upon the death of a joint tenant, the surviving joint tenant receives the decedent's interest by
operation of law.

D. Qualified joint tenancies cannot be severed without mutual consent.

Answer: C

QUESTION NO: 27

Which of the following statements concerning the methods of valuing a closely held business for federal
estate tax purposes is (are) correct?
1. The capitalization-of-adjusted-earnings method uses a capitalization rate that varies inversely with
the degree of risk and rate of return.

2. The adjusted-book value method involves adjusting the asset components of a business to an
approximate fair market value for each component.

A. Both 1 and 2

B. 1 only

C. 2 only

D. Neither 1 nor 2

Answer: A

QUESTION NO: 28

Alan, a widower, is a retired executive with substantial assets. He wishes to provide for the financial
security of his two grandchildren since their father, Alan's son, has always managed money poorly. This
year Alan would like each grandchild to receive a substantial gift. Which of the following statements
concerning the generation-skipping transfer tax (GSTT) on these gifts is (are) correct?

1. Federal estate or gift tax will not be imposed if the gift is otherwise subject to the GSTT.

2. Assuming no prior gifts, Alan can gift a cumulative total of (not including the annual exclusion)
$1.5 million to his grandchildren without the imposition of the GSTT.

A. 2 only

B. Neither 1 nor 2
C. Both 1 and 2

D. 1 only

Answer: A

QUESTION NO: 29

Which of the following statements concerning marital transfers to a non-U.S. citizen spouse is (are)
correct?

1. A marital deduction is automatically available as long as property is transferred outright to the non-
citizen spouse.

2. A marital deduction is automatically available if the transferor-decedent spouse is a U. S. citizen.

A. 2 only

B. Both 1 and 2

C. Neither 1 nor2

D. 1 only

Answer: C

QUESTION NO: 30

Examples of terminable interests that are nondeductible for the federal estate tax marital deduction
include which of the following?
1. A property interest that would pass to someone else if the surviving spouse remarries

2. A life estate in property that would pass to another person at the death of the surviving spouse

A. Both 1 and 2

B.1 only

C. Neither 1 nor 2

D. 2 only

Answer: A

QUESTION NO: 31

Which of the following statements concerning a power of appointment is (are) correct?

1. A power of appointment can be designed to restrict the donee's right to appoint the property in
favor of a specific class of recipients.

2. A power of appointment can be designed to restrict the time period during which the donee
may exercise the power.

A. Neither 1 nor 2

B. 2 only

C. Both 1 and 2

D. 1 only
Answer: C

QUESTION NO: 32

In addition to substantial probate assets, a married man with two minor children has a $1,000,000
ordinary life insurance policy payable to his estate. He wants to make certain that if he predeceases his
wife the death proceeds will be available to provide income for his wife during her lifetime and to
provide for their two children after her death. He would like the policy and/or its death proceeds to be
as free of federal gift and estate taxes as possible with respect to both him and his wife. Which of the
following courses of action would best accomplish these objectives?

A. Designate his wife as beneficiary and she will establish a testamentary trust in her will to receive the
proceeds at her subsequent death

B. Assign the policy to an irrevocable inter vivos trust with five and five powers and designate the
trustee to receive the death proceeds

C. Establish a revocable inter vivos trust and designate the trustee to receive death proceeds

D. Assign the policy to his wife who will establish a revocable inter vivos trust to receive the death
proceeds

Answer: B

QUESTION NO: 33

All the following statements concerning an estate for a term of years are correct EXCEPT:

A. An interest may extend beyond the lifetime of the grantor.

B. The tenant may transfer the property at the end of the term of his interest.

C. It is an interest in property established for a specific duration.

D. The tenant has the right to possess the property during the term of his interest.
Answer: B

QUESTION NO: 34

Which of the following statements concerning pooled-income funds is (are) correct?

1. A pooled income fund is similar to a mutual fund maintained by a qualified charity.

2. It is an irrevocable arrangement in which the remainder interest passes to charity.

A. Neither 1 nor 2

B. Both 1 and 2

C. 1 only

D. 2 only

Answer: B

QUESTION NO: 35

A father deeded a house as a gift to his daughter in 1990 but retained the right to live in it until his
death. He died this year while still living in the house. The following are relevant facts:

The father bought the property in 1980 for $130,000. The fair market value of the property when the
gift was made in 1990 was $150,000. The father filed a timely gift tax return but paid no gift tax because
of the applicable credit amount. The fair market value of the property at the father's death was
$220,000. The daughter sold the property 3 months after her father's death for $220,000. She had a
gain of?

A. $140,000

B. $220,000

C. $120,000

D. 0

Answer: D

QUESTION NO: 36

All the following statements concerning a power of appointment are correct EXCEPT:

A. The donee of a power of appointment might be restricted with respect to the time the power can be
exercised.

B. The appointee of a power of appointment is the party who exercises the power.

C. Failure to exercise a power of appointment is known as a lapse of the power.

D. The possible recipients of property after the exercise of a power of appointment depend on the terms
of the power.

Answer: B

QUESTION NO: 37

The failure of an individual to have a will can result in all the following EXCEPT:
A. A surviving spouse receives only his or her elective share.

B. Unnecessary death taxes may be imposed.

C. Testamentary gifts to charity cannot be made.

D. The decedent's state of domicile might receive the property left by the decedent.

Answer: A

QUESTION NO: 38

A father is considering giving his daughter a gift. For tax planning purposes, the father should give his
daughter which of the following?

A. Raw land that cost him $10,000, its present fair market value, but which has a substantial potential
for appreciation

B. Real estate that cost him $40,000 and is now worth $120,000, subject to a $110,000 mortgage

C. Stock that cost him $10,000 and which now has a fair market value of $20,000

D. A bond that cost him $15,000 and is now worth $10,000

Answer: A

QUESTION NO: 39

All the following are proper actions on the part of a trustee EXCEPT

A. placing cash from the sale of securities in a non-interest bearing checking account for an extended
period of time
B. investing trust assets in speculative securities in accordance with the provisions of the trust
instrument

C. purchasing securities in good faith from a third party just prior to a sharp decrease in their value

D. purchasing assets for personal use from the trust at their fair market value with the approval of all
beneficiaries

Answer: A

QUESTION NO: 40

All the following trust provisions avoid causing the inclusion of an irrevocable life insurance trust in an
insured's gross estate EXCEPT

A. a trust beneficiary's power to withdraw contributions to the trust

B. a trustee's power to loan assets to the estate at the trustee's discretion

C. a trustee's power to pay estate expenses

D. a trustee's power to purchase assets from the estate at the trustee's discretion

Answer: C

QUESTION NO: 41

For estate tax purposes which of the following is (are) a form of charitable gifts that may qualify for the
charitable deduction?

1. Split gifts
2. Charitable remainder trusts

A. Neither 1 nor 2

B. 1 only

C. 2 only

D. Both 1 and 2

Answer: D

QUESTION NO: 42

A single man with substantial assets and income is supporting his 80-year-old partially senile mother
with monthly cash gifts. He is trying to find a practical way to support his mother while at the same time
saving federal gift and income taxes without giving up ultimate control of any assets. Which of the
following courses of action will best accomplish these objectives?

A. Make her an interest free loan with a principal amount large enough to produce sufficient income for
her support when invested in corporate bonds

B. Purchase corporate bonds that pay interest in an amount sufficient for her to support herself and
assign the interest payments to her

C. Make her annual gifts of enough interest income from the tax free municipal bonds in his portfolio so
that she will be able to support herself

D. Make her a gift of enough corporate bonds from his portfolio so that she will be able to support
herself from the interest payments

Answer: C
QUESTION NO: 43

All the following statements concerning the generation-skipping transfer tax (GSTT) are correct EXCEPT:

A. The tax is imposed according to a graduated rate schedule similar to the federal estate and gift tax
rates.

B. The tax may be imposed on gifts in trust to grandchildren.

C. The tax may be imposed on direct gifts to grandchildren.

D. All donors have a cumulative $1.5 million exemption against generation-skipping transfers.

Answer: A

QUESTION NO: 44

In which of the following situations will the grantor be taxed on income from trust property.

1. The grantor of a trust gives one of the trust beneficiaries the right to add or delete beneficiaries.

2. An adverse party to the grantor holds the power to determine the timing of trust distributions to
the beneficiaries.

A. Neither 1 nor 2

B. 2 only

C. 1 only

D. Both 1 and 2

Answer: A
QUESTION NO: 45

In which of the following situations will the grantor be taxed on income from trust property.

1. The grantor of a trust gives one of the trust beneficiaries the right to add or delete beneficiaries.

2. An adverse party to the grantor holds the power to determine the timing of trust distributions to
the beneficiaries.

A. 2 only

B. 1 only

C. Neither 1 nor 2

D. Both 1 and 2

Answer: C

QUESTION NO: 46

In which of the following estates would it normally be most desirable to provide for a federal estate tax
marital deduction for approximately one half of the property?

A. In the estate of the spouse with the bulk of the family wealth when only one spouse has substantial
wealth

B. In the estate of the first spouse to die, regardless of the relative wealth of each spouse

C. In the estate of the first spouse to die when neither spouse has a substantial estate
D. In the estate of the spouse who does not have the bulk of the family wealth when only one spouse
has substantial wealth

Answer: A

QUESTION NO: 47

Which of the following statements concerning the federal income taxation of estates is (are) correct?

1. An estate is entitled to a personal exemption of $300 and a standard deduction.

2. An estate is entitled to a tax deduction for amounts of income distributed.

A. 2 only

B. Both 1 and 2

C. 1 only

D. Neither 1 nor 2

Answer: A

QUESTION NO: 48

A married man died this year leaving a gross estate of $2,700,000. Some additional facts concerning his
estate are:

1. Administration expenses and debts $300,000


2. Marital deduction 800,000
3. Applicable credit amount (2005) 555,800
4. Applicable exclusion amount (2005) 1,500,000
5. State death taxes payable 17,700

Under the Unified Rate Schedule for computing estate taxes if the amount with respect to which the
tentative tax to be computed is over $1,000,000 but not over $1,250,000, the tentative tax is $345,800,
plus 41 percent of the excess of such amount over $1,000,000. If the amount is over $1,250,000 but not
over $1,500,000, the tentative tax is then $448,300, plus 43 percent of the excess of such amount over
$1,250,000. If the amount is over $1,500,000 but not over $2,000,000 the tentative tax is then $555,800
plus 45% of the excess of such amount over $1,500,000. Based on these facts, the net federal tax
payable is

A. $47,065

B. $42,865

C. 0

D. $37,035

Answer: D

QUESTION NO: 49

Which of the following statements concerning revocable trusts is correct?

A. Property of a revocable trust will be included in the grantor's probate property.

B. A transfer of income-producing property to a revocable trust will result in favorable income tax
treatment for the grantor.

C. A transfer of property to a revocable trust is treated as a completed gift.

D. The transfer of property to a revocable trust is typically motivated by nontax reasons.


Answer: D

QUESTION NO: 50

A married man is the sole owner of a small business with an estate tax value of $500,000. In addition, he
and his wife own an office building as joint tenants with right of survivorship which they purchased five
years ago. The building has an estate tax value of $1,500,000. They are considering dissolving the joint
tenancy and retitling the building in the name of the husband as sole owner. All the following
statements concerning this action are correct EXCEPT:

A. At the husband's death, his heirs would get a fully stepped-up tax basis for the property.

B. At the husband's death, it would be easier to qualify his estate for an IRC Section 303 stock
redemption of his business interest.

C. If the husband dies first and leaves the office building outright to his wife, there would be no federal
estate tax attributed to its inclusion in his gross estate.

D. If the husband dies first, the probate costs of his estate could be increased.

Answer: B

QUESTION NO: 51

Which of the following statements concerning the inclusion in a decedent-employee's gross estate of a
lump-sum distribution from a qualified retirement plan to a beneficiary other than the employee's
estate is (are) correct?

1. Lump-sum distributions of payments attributable to the employer's contributions are excluded from
the gross estate.
2. Lump-sum distributions of payments attributable to the decedent-employee's contributions are
excluded from the gross estate.

A. Neither 1 nor 2

B. Both 1 and 2

C. 1 only

D. 2 only

Answer: A

QUESTION NO: 52

Which of the following statements concerning charitable guaranteed annuity interests is (are) correct?

1. To qualify for an estate tax charitable deduction, guaranteed annuity interests must be made in trust.

2. These interests refer to the charity right to receive a determinable income amount at least annually
for a specific term or life (lives) or one or more noncharitable beneficiaries.

A. Both 1 and 2

B. Neither 1 nor 2

C. 2 only

D. 1 only

Answer: C
QUESTION NO: 53

The Decedent, T, died this year. The facts concerning T estate are:

 Gross estate $2,700,000


 Marital deduction 900,000
 Charitable deduction 110,000
 Gifts made after 1976 130,000
 State death taxes payable 165,000

What is T taxable estate?

A. $1,395,000

B. $1,285,000

C. $1,655,000

D. $1,525,000

Answer: D

QUESTION NO: 54

All the following are grounds for contesting a will EXCEPT:

A. The testator executed a later valid will.

B. The instrument is a forgery.

C. The widow was bequeathed less than her intestate share.

D. The testator did not have testamentary capacity.


Answer: C

QUESTION NO: 55

A taxable gift has been made in which of the following situations?

1. A father manages his disabled son's business for a year without compensation since a replacement
manager would have cost $25,000.

2. A father verbally promises his 21-year-old daughter that he will give her his antique Mercedes when
she graduates from college next year.

A. 2 only

B. Neither 1 nor 2

C. Both 1 and 2

D. 1 only

Answer: B

QUESTION NO: 56

All the following transfers are subject to the generation-skipping transfer tax (GSTT) EXCEPT:

A. A distribution to a grandchild from a sprinkle trust created by a grandparent to benefit both skip and
non-skip beneficiaries.
B. A direct cash gift of $50,000 from a grandparent to his grandchild if such grandchild's parents are still
alive.

C. A direct cash payment of $28,000 from a grandparent to a private prep school to cover the tuition
costs for her grandchild.

D. A termination of a trust at the death of the nonskip life income beneficiary with the remainder
distributed solely to skip persons.

Answer: C

QUESTION NO: 57

Which of the following statements concerning the valuation of intangible personal property in the gross
estate of a decedent is correct?

A. If there were no trades of a listed common stock on the date of the stockholder's death, the stock's
value is based on its average daily price for the previous month prior to the shareholder's death.

B. Certain U.S. Treasury bonds that are used to pay federal estate taxes at par are valued at their market
price on the date of death of the owner.

C. When a minority stockholder in a closely held corporation dies, his stock is valued on the basis of the
"blockage" rule.

D. Valuing closely held stock requires the consideration of several factors outlined by IRS rulings.

Answer: D

QUESTION NO: 58

A father and son have been farming land owned by the father for the past 12 years. Just prior to his
death, the father was offered $1,200,000 for his farm because of its possible use as a shopping center.
The son would like to continue to farm the land if it can be included in his father's estate at its current
use value. Additional facts are:

1.Average annual gross rentals from nearby farms of similar acreage are $56,000.

2.Average annual state and local real estate taxes on the farm are $8,000.

3.The interest rate for loans from the Federal Land Bank is 8 percent.

For federal estate tax purposes, the farm method valuation formula would result in a current use value
for the farm of

A. $700,000

B. $820,000

C. $600,000

D. $500,000

Answer: C

QUESTION NO: 59

Which of the following statements concerning the gift or estate tax charitable deduction is (are) correct?

1. If the donor retains an interest in property contributed to a qualified charity during lifetime, the value
of the property may be included in the donor gross estate.

2. An estate tax charitable deduction is allowed for the full value of property transferred to a qualified
charity but only if the property is included in the donor gross estate.
A. 2 only

B. 1 only

C. Both 1 and 2

D. Neither 1 nor 2

Answer: C

QUESTION NO: 60

A wife makes outright gifts of $66,000 this year to her son, and her husband agrees to split the gifts with
her. Which of the following correctly states the amount of the taxable gifts?

A. Wife $22,000, husband $22,000

B. Wife $44,000, husband 0

C. Wife $12,000, husband $32,000

D. Wife $32,000, husband $32,000

Answer: A

QUESTION NO: 61

Ignoring the annual per-donee exclusion, all the following transfers are gifts for federal gift tax purposes
EXCEPT:

A. An individual gratuitously performs valuable services for the benefit of a close friend.

B. A grandmother reimburses her grandson for his college tuition costs.


C. A creditor cancels the promissory note of a friend who recently became unemployed.

D. A father lends his daughter a large sum of money interest free for a period of 2 years.

Answer: A

QUESTION NO: 62

A father wants to accumulate funds for his 12-year-old son's college education. On the advice of his
attorney, the father establishes an IRC Section 2503(c) trust and funds it with annual gifts. All the
following statements concerning this arrangement are correct EXCEPT:

A. In the event of the son's death prior to age 21, trust assets must either be payable to the son's estate
or be subject to a general power of appointment held by the son.

B. The father's annual gift tax exclusion must be reduced by any amount used to pay college tuition
costs.

C. The trust must be irrevocable.

D. Any accumulated income and all trust principal must be available for distribution to the son when he
attains age 21.

Answer: B

QUESTION NO: 63

The executor of an estate has a choice of waiving the executor's fee. Factors that should be considered
by the executor in making this choice include all the following EXCEPT

A. the estate tax bracket of the estate

B. the income tax brackets of the other beneficiaries


C. whether the executor is otherwise a beneficiary of the estate

D. the income tax bracket of the executor

Answer: B

QUESTION NO: 64

All the following statements concerning an entity-purchase buy-sell agreement for a partnership are
correct EXCEPT:

A. The partnership, in effect, liquidates the interest held by the decedent-partner's estate.

B. The partnership makes payments to the decedent-partner's estate to liquidate the partnership
interest held by the estate.

C. It is the surviving partners who purchase the decedent's business interest so that the partnership can
afford to pay the decedent's estate.

D. Both the partners and the partnership are parties to the agreement that provides for business
continuation.

Answer: C

QUESTION NO: 65

A married man has two adult sons. His entire estate is in excess of $1,500,000 and consists entirely of
probate assets. He wants to make certain that if he predeceases his wife she will receive all estate
income as long as she lives, and the assets remaining at her death will pass equally to their two sons. He
wants to pass all assets to this wife and sons as free of federal estate taxes as possible. To best
accomplish these objectives, the man should include which of the following estate plans in his will?
A. Establish a marital deduction trust with a general power of appointment for half his estate and place
the remainder in a QTIP trust

B. Establish a QTIP trust for half his estate and bequeath the remainder to his wife

C. Establish a QTIP trust for his entire estate

D. Establish a bypass trust equal to the applicable exclusion amount and place the remainder of his
estate in a QTIP trust

Answer: D

QUESTION NO: 66

Which of the following statements concerning the taxation of estates and trusts is (are) correct?

1. They are taxed similarly to partnership entities.

2. They are taxed on distributable net income (DNI) that is retained.

A. 1 only

B. Both 1 and 2

C. Neither 1 nor 2

D. 2 only

Answer: D

QUESTION NO: 67
All the following assets owned by a decedent pass by operation of law or by contract EXCEPT

A. death benefits of a pension plan payable to a named beneficiary

B. life insurance payable to a named beneficiary

C. probate assets passing under the terms of a valid will

D. jointly owned real estate passing to the surviving joint tenant

Answer: C

QUESTION NO: 68

A woman is the income beneficiary of an irrevocable trust. All the following powers held by her will
cause all the assets in the trust to be includible in her gross estate for federal estate tax purposes
EXCEPT

A. the testamentary special or limited power to direct the trustee to distribute trust assets to her
children

B. the testamentary power to direct the trustee to use trust assets to pay her estate taxes

C. the power to direct the trustee to distribute trust corpus to her

D. the testamentary power to direct the trustee to pay trust assets to her estate

Answer: A

QUESTION NO: 69

All the following statements concerning guardians for minors are correct EXCEPT:
A. A guardian has equitable title to the property he administers for the minor.

B. A guardian named in a deceased parent's will is not necessarily binding on the court.

C. A guardian of the person of a minor may not necessarily be the guardian of the minor's property.

D. A special guardian can be appointed by the court to protect a minor's rights in a legal proceeding.

Answer: A

QUESTION NO: 70

Which of the following are ways of passing property from a deceased spouse to a surviving spouse so
that the property will qualify for the federal estate tax marital deduction?

1. When the surviving spouse receives the property by electing to take against the deceased spouse's
will

2. When the surviving spouse receives the property as a consequence of the qualified disclaimer of
another beneficiary

A. Both 1 and 2

B. Neither 1 nor 2

C. 1 only

D. 2 only

Answer: A

QUESTION NO: 71
Which of the following statements concerning an installment sale is (are) correct?

1. Some portion of the purchase price must be paid in any one taxable year other than the year of sale.

2. Installments due after the seller's death are excludible from the seller's gross estate.

A. Both 1 and 2

B. 1 only

C. 2 only

D. Neither 1 nor 2

Answer: B

QUESTION NO: 72

On January 1, 2004 a father gave his daughter a $150,000 straight (ordinary) life insurance policy on his
life. Premiums are paid annually. The pertinent facts about the policy are:

Date of issue: July 1, 1992

Premium paid on July 1, 2003 $2,400

Terminal reserve on July 1, 2003 15,000

Terminal reserve on July 1, 2004 18,000


What is the value of the policy for federal gift tax purposes?

A. $16,200

B. $150,000

C. $17,700

D. $17,400

Answer: C

QUESTION NO: 73

Which of the following statements concerning a general power of appointment is (are) correct?

1. A general power of appointment is a power over property so broad that it approaches actual
ownership or control over the property subject to the power.

2. The property subject to a general power of appointment will be includible in the gross estate of the
holder of the power if held until death.

A. Both 1 and 2

B. Neither 1 nor 2

C. 2 only

D. 1 only

Answer: A
QUESTION NO: 74

Which of the following statements concerning both estates and complex trusts is correct?

A. Both are required to file income tax returns.

B. Both are monitored by the courts.

C. Both must have more than one beneficiary.

D. Both come into being by operation of law.

Answer: A

QUESTION NO: 75

Which of the following terms applies to the blending together of separate and community properties of
spouses in community-property states?

A. Proration

B. Inception

C. Commingling

D. Transmutation

Answer: C

QUESTION NO: 76
In addition to substantial probate assets, a married man with two minor children has a $1,000,000
ordinary life insurance policy payable to his estate. He wants to make certain that if he predeceases his
wife the death proceeds will be available to provide income for his wife during her lifetime and to
provide for their two children after her death. He would like the policy and/or its death proceeds to be
as free of federal gift and estate taxes as possible with respect to both him and his wife. Which of the
following courses of action would best accomplish these objectives?

A. Assign the policy to an irrevocable inter vivos trust with five and five powers and designate the
trustee to receive the death proceeds

B. Assign the policy to his wife who will establish a revocable inter vivos trust to receive the death
proceeds

C. Establish a revocable inter vivos trust and designate the trustee to receive death proceeds

D. Designate his wife as beneficiary and she will establish a testamentary trust in her will to receive the
proceeds at her subsequent death

Answer: A

QUESTION NO: 77

Which of the following types of partial interests in property may be allowed a charitable deduction for
estate tax purposes?

1. A remainder interest in the donor vacation home

2. A testamentary gift of a percentage of a decedent entire interest in property held in trust

A. Neither 1 nor 2

B. Both 1 and 2

C. 1 only
D. 2 only

Answer: C

QUESTION NO: 78

To determine whether a taxable gift has been made, the IRS focuses on all the following factors EXCEPT:

A. Was the value of the gift property in excess of the annual per-donee exclusion?

B. Was the property transferred for less than an adequate and full consideration in money or money's
worth?

C. Was the transferred property real property or personal property?

D. Did the donor absolutely, irrevocably, and currently divest himself of dominion and control over the
property?

Answer: C

QUESTION NO: 79

Which of the following statements concerning the so-called "kiddie-tax" on unearned income of children
under age 14 is (are) correct?

1. The rules apply to earned income of the children.

2. The rules apply to trust income received by a child under age 14 only if the trust was established by
the child's parents.
A. Both 1 and 2

B. Neither 1 nor 2

C. 1 only

D. 2 only

Answer: B

QUESTION NO: 80

Which of the following life insurance settlement options will qualify for the federal estate tax marital
deduction?

1. Proceeds left to the surviving spouse under the interest option, with interest payable to the
surviving spouse who has the unrestricted right to withdraw proceeds and with any proceeds not
withdrawn payable equally to her children per stirpes

2. Proceeds left to the surviving spouse under an installment option, with any installments remaining
at her death to be commuted and paid to her estate

A. Both 1 and 2

B. 1 only

C. 2 only

D. Neither 1 nor 2

Answer: A
QUESTION NO: 81

If a grantor establishes an irrevocable trust, the income of the trust will be taxed to the grantor if it is
used to pay premiums for life insurance on the life of

A. the father of the grantor

B. a child of the grantor

C. the spouse of the grantor

D. a grandchild of the grantor

Answer: C

QUESTION NO: 82

All the following transfers are subject to the generation-skipping transfer tax (GSTT) EXCEPT:

A. A direct cash payment of $28,000 from a grandparent to a private prep school to cover the tuition
costs for her grandchild.

B. A distribution to a grandchild from a sprinkle trust created by a grandparent to benefit both skip and
non-skip beneficiaries.

C. A termination of a trust at the death of the nonskip life income beneficiary with the remainder
distributed solely to skip persons.

D. A direct cash gift of $50,000 from a grandparent to his grandchild if such grandchild's parents are still
alive.

Answer: A
QUESTION NO: 83

All the following statements concerning property ownership by a married couple residing in a
community-property state are correct EXCEPT:

A. Income earned by one spouse becomes community property.

B. All property that is not separate property is community property.

C. Property inherited during the marriage is the separate property of the spouse who inherited it.

D. Community property loses its identity when a community-property couple moves to a common-law
state.

Answer: D

QUESTION NO: 84

A wife owns a $100,000 life insurance policy on her husband's life. She has named her son the revocable
beneficiary. Which of the following statements concerning the life insurance is (are) correct?

1. At the husband's death, the interpolated terminal reserve of the policy is a gift to the son.

2. The annual increase in the cash value is a gift to the son.

A. Neither 1 nor 2

B. 1 only

C. 2 only

D. Both 1 and 2

Answer: A
QUESTION NO: 85

Which of the following statements concerning state death tax exemptions and tax rates for classes of
estate beneficiaries is (are) correct?

1. Exemptions are determined by the closeness of the beneficiary's blood relationship to the decedent.

2. Closest relatives receive the lowest tax rates and lowest exemption amounts.

A. 1 only

B. Neither 1 nor 2

C. Both 1 and 2

D. 2 only

Answer: A

QUESTION NO: 86

A father plans to create a trust for the benefit of his 22-year-old son and wishes to take advantage of the
gift tax annual exclusion. He has named a bank as trustee. Which of the following trust provisions would
cause the gifts to be ineligible to qualify for the gift tax annual exclusion?

1. The trust income is to be paid to the son or accumulated at the discretion of the trustee.

2. The income is to be accumulated until the son reaches age 32 when all accumulated income and
principal are to be distributed to him.
A. 1 only

B. Both 1 and 2

C. Neither 1 nor 2

D. 2 only

Answer: B

QUESTION NO: 87

Income earned but unpaid at the time of a decedent's death is deemed to be income in respect of a
decedent (IRD). All the following statements concerning IRD are correct EXCEPT:

A. The income must be reported on the decedent's final federal income tax return.

B. IRD includes sales commissions earned prior to the decedent's death and paid to the estate according
to the intestacy laws.

C. The income is taxable to the person or entity receiving it.

D. The income may be included on both the estate tax return and the estate income tax return with a
corresponding deduction.

Answer: A

QUESTION NO: 88

Which of the following statements concerning a grantor-retained annuity trust (GRAT) is correct?
A. The transfer of property to the trust becomes a completed gift for gift tax purposes only at the
termination of the grantor's retained interest term.

B. The trust is used as a device for shifting income tax on the current trust income to the
remainderperson(s).

C. If the grantor dies before the retained interest term ends, the estate tax benefits are reduced.

D. The grantor retains control of the trust property until revocation or death.

Answer: C

QUESTION NO: 89

A father died leaving his property equally to his wealthy son and his poor daughter. The son wishes to
disclaim his share of the inheritance so that it will pass to his sister without his incurring any gift tax
liability. In this situation, all the following acts on the part of the son are required EXCEPT:

A. He must not have received any part of his inheritance or any income from it prior to his refusal to
accept it.

B. His refusal to accept the inheritance must be received by the executor of his father's estate within 9
months of his father's death.

C. His refusal to accept the inheritance must be in writing.

D. His refusal to accept the inheritance must direct specifically that his sister is to receive it instead.

Answer: D

QUESTION NO: 90

Mr. Barlow died early this year. Under the terms of his will he left all his real estate and tangible
personal property to his son. All the remainder of his probate estate was left to his wife, Mrs. Barlow.
The following is a list of Mr. Barlow's probate assets and their fair market values at the time of his death:
 Commercial real estate $150,000
 Furniture and fixtures 75,000
 Listed common stock 300,000
 Other securities 200,000

In addition, Mr. Barlow also owned a $100,000 life insurance policy on his life with Mrs. Barlow
designated as beneficiary. Based on this information, what is the amount of property in Mr. Barlow's
estate qualifying for the federal estate tax marital deduction?

A. $825,000

B. $500,000

C. $400,000

D. $600,000

Answer: D

QUESTION NO: 91

The executor of an estate has a choice of waiving the executor's fee. Factors that should be considered
by the executor in making this choice include all the following EXCEPT

A. the estate tax bracket of the estate

B. the income tax brackets of the other beneficiaries

C. whether the executor is otherwise a beneficiary of the estate

D. the income tax bracket of the executor

Answer: B
QUESTION NO: 92

All the following statements concerning property ownership by a married couple residing in a
community-property state are correct EXCEPT:

A. All property that is not separate property is community property.

B. Property inherited during the marriage is the separate property of the spouse who inherited it.

C. Community property loses its identity when a community-property couple moves to a common-law
state.

D. Income earned by one spouse becomes community property.

Answer: C

QUESTION NO: 93

In addition to substantial probate assets, a married man with two minor children has a $1,000,000
ordinary life insurance policy payable to his estate. He wants to make certain that if he predeceases his
wife the death proceeds will be available to provide income for his wife during her lifetime and to
provide for their two children after her death. He would like the policy and/or its death proceeds to be
as free of federal gift and estate taxes as possible with respect to both him and his wife. Which of the
following courses of action would best accomplish these objectives?

A. Assign the policy to his wife who will establish a revocable inter vivos trust to receive the death
proceeds

B. Assign the policy to an irrevocable inter vivos trust with five and five powers and designate the
trustee to receive the death proceeds

C. Designate his wife as beneficiary and she will establish a testamentary trust in her will to receive the
proceeds at her subsequent death
D. Establish a revocable inter vivos trust and designate the trustee to receive death proceeds

Answer: B

QUESTION NO: 94

Ignoring the annual per-donee exclusion, which of the following transfers is a gift for federal gift tax
purposes?

A. A father promises to buy his daughter a condominium when she finishes college.

B. An individual gratuitously performs valuable services for the benefit of a close friend.

C. A grandmother pays her grandson's $30,000 tuition at an Ivy League university.

D. A creditor cancels the promissory note of a recently unemployed friend as a charitable gesture.

Answer: D

QUESTION NO: 95

Limited interests in property include all the following EXCEPT

A. life estates

B. remainder interests

C. fee simple estates

D. reversionary interests

Answer: C
QUESTION NO: 96

John plans to transfer his life insurance policy to an irrevocable trust for the benefit of his 19-year-old
daughter, Jane. Which of the following conditions will enable the gift to qualify for the annual exclusion?

1. Jane is the irrevocable beneficiary of the life insurance trust but cannot withdraw from the trust until
the death benefits are paid.

2. Jane is given "Crummey" demand powers permitting the withdrawal at her discretion of the annual
additions to the trust.

A. 1 only

B. Neither 1 nor 2

C. Both 1 and 2

D. 2 only

Answer: D

QUESTION NO: 97

Which of the following statements concerning charitable remainder annuity trusts is correct?

A. It provides a fixed annuity income interest to a qualified charity.

B. The donor receives an estate tax charitable deduction for the value of the remainder interest.

C. The term of charitable remainder annuity arrangements is limited to 10 years.

D. The interest paid to the charity must be paid at least quarterly.


Answer: B

QUESTION NO: 98

Important factors in assessing liquidity needs in estate planning include which of the following?

1. The types of assets that comprise the estate

2. The date of drafting the will

A. 1 only

B. Both 1 and 2

C. 2 only

D. Neither 1 nor 2

Answer: A

QUESTION NO: 99

Which of the following statements concerning state death taxes is correct?

A. A state inheritance tax is imposed on the right of the deceased to leave property to heirs.

B. A state estate tax is imposed on the right of heirs to receive property from the deceased.

C. State estate and inheritance taxes are generally imposed at the same rate regardless of the
relationship of the deceased to the beneficiary.
D. A deduction for the full amount of state death taxes paid by a decedent estate is allowed on a
decedent federal estate tax return.

Answer: D

QUESTION NO: 100

Which of the following transfers will be successful in removing property from a grantor's gross estate?

1. A grantor's transfer of property to a revocable trust if the grantor lives three years after the transfer.

2. A grantor's transfer of a personal residence to a qualified personal residence trust if the grantor
survives the retained interest term.

A. Neither 1 nor 2

B. 1 only

C. 2 only

D. Both 1 and 2

Answer: C

QUESTION NO: 101

Which of the following statements concerning wills is (are) correct?

1. A will should be reviewed periodically to assure that the property owner's most recent intentions are
honored at death.
2. Once signed, a will's provisions may not be changed without the consent of all the beneficiaries
under the will.

A. Both 1 and 2

B. 1 only

C. 2 only

D. Neither 1 nor 2

Answer: B

QUESTION NO: 102

All the following transfers are subject to the generation-skipping transfer tax (GSTT) EXCEPT:

A. A direct cash payment of $28,000 from a grandparent to a private prep school to cover the tuition
costs for her grandchild.

B. A termination of a trust at the death of the nonskip life income beneficiary with the remainder
distributed solely to skip persons.

C. A distribution to a grandchild from a sprinkle trust created by a grandparent to benefit both skip and
non-skip beneficiaries.

D. A direct cash gift of $50,000 from a grandparent to his grandchild if such grandchild's parents are still
alive.

Answer: A
QUESTION NO: 103

Tax benefits of making lifetime gifts in excess of the gift tax annual exclusion include all the following
EXCEPT:

A. Gift taxes are payable at the same tax rate as estate taxes.

B. Income taxes can be saved if a high-income donor gives income-producing property to a low-income
donee.

C. The gift tax paid on a gift made more than 3 years prior to the death of the donor avoids inclusion the
donor's gross estate.

D. Appreciation in the value of a gift of real property after the date of the gift increases the donor's
federal estate tax liability.

Answer: D

QUESTION NO: 104

A man died in February of this year. Last year, when he learned that he has a terminal illness, he
immediately made the following gifts and filed the required gift tax return:

Fair Market Value

 Gift of listed stock to a


 qualified charity $150,000
 Gift of listed bonds to his wife 200,000
 Gift of a boat to his son 5,000
 Gift of a sports car to his daughter 5,000

What amount must be brought back to the man's estate as an adjusted taxable gift in the calculation of
his federal estate taxes?
A. $190,000

B. $360,000

C. $150,000

D. 0

Answer: D

QUESTION NO: 105

On January 1, 2004 a father gave his daughter a $50,000 straight (ordinary) life insurance policy on his
life. Premiums are paid annually. The pertinent facts about the policy are:

Date of issue: July 1, 1992

 Premium paid on July 1, 2003 $800


 Terminal reserve on July 1, 2003 5,000
 Terminal reserve on July 1, 2004 6,000

What is the value of the policy for federal gift tax purposes?

A. $5,800

B. $50,000

C. $5,900

D. $5,400

Answer: C
QUESTION NO: 106

Which of the following factors is (are) used to make a choice between having an entity-purchase or a
cross-purchase partnership buy-sell agreement?

1. The number of partners

2. The divergence of the partners' business knowledge and skills

A. Both 1 and 2

B. 2 only

C. Neither 1 nor 2

D. 1 only

Answer: D

QUESTION NO: 107

A man is planning to establish and fund an irrevocable trust for the benefit of his two sons, ages 19 and
22, and plans to give the trustee power to sprinkle trust income. From the standpoint of providing
federal income, gift, and estate tax savings, which of the following would be a suitable trustee?

1. The grantor of the trust

2. The grantor's 22-year-old son

A. Neither 1 nor 2
B. 1 only

C. Both 1 and 2

D. 2 only

Answer: A

QUESTION NO: 108

Which of the following life insurance settlement options will qualify for the federal estate tax marital
deduction?

1. Proceeds left to the surviving spouse under the interest option, with interest payable to the surviving
spouse who has the unrestricted right to withdraw proceeds and with any proceeds not withdrawn
payable equally to her children per stirpes

2. Proceeds left to the surviving spouse under an installment option, with any installments remaining at
her death to be commuted and paid to her estate

A. Both 1 and 2

B. Neither 1 nor 2

C. 2 only

D. 1 only

Answer: A

QUESTION NO: 109


A father bought stock for $100,000 and gave it to his son when it was worth $300,000. The father paid
no gift tax on the transfer. When the son sold the property 2 years after the gift, his income tax basis
was

A. $300,000

B. $200,000

C. $100,000

D. 0

Answer: C

QUESTION NO: 110

Transactions involving a taxable gift include which of the following?

1. A father bought real estate, paid the entire $180,000 purchase price, and titled it jointly with his son
with right of survivorship.

2. A father deposited $50,000 in a bank account titled jointly with his daughter with right of survivorship
and died before any funds were withdrawn from the account.

A. 1 only

B. 2 only

C. Neither 1 nor 2

D. Both 1 and 2

Answer: A
QUESTION NO: 111

Which of the following statements concerning the inclusion in a decedent-employee's gross estate of a
lump-sum distribution from a qualified retirement plan to a beneficiary other than the employee's
estate is (are) correct?

1. Lump-sum distributions of payments attributable to the employer's contributions are excluded from
the gross estate.

2. Lump-sum distributions of payments attributable to the decedent-employee's contributions are


excluded from the gross estate.

A. Both 1 and 2

B. 1 only

C. 2 only

D. Neither 1 nor 2

Answer: D

QUESTION NO: 112

All the following are steps in calculating a decedent's maximum estate tax marital deduction EXCEPT:

A. Subtract the allowable expenses and debts to determine the adjusted gross estate.

B. Subtract the applicable exclusion amount available in the year of the decedent's death.

C. Compute the decedent's gross estate.


D. Determine the net amount of property in the gross estate that passes to the surviving spouse in a
manner qualifying for the marital deduction.

Answer: B

QUESTION NO: 113

Which of the following statements concerning the taxation of estates and trusts is correct?

A. They are taxed on distributable net income that is retained.

B. They are taxed as if they were partnerships.

C. They are entitled to the standard deduction in determining tax.

D. They are taxed as if they were corporations.

Answer: A

QUESTION NO: 114

All the following statements concerning filing the federal estate tax return are correct EXCEPT:

A. An automatic one-year extension for filing the estate tax return is granted when the decedent dies
overseas.

B. The estate tax return must be filed within 9 months of death unless an extension is granted by the
IRS.

C. An extension to file the estate tax return must be received and granted by the IRS before the time for
filing the return expires.
D. For persons dying this year, an estate tax return must be filed for gross estates plus adjusted taxable
gifts that exceed $1.5 million.

Answer: A

QUESTION NO: 115

Which of the following statements concerning the inclusion in a decedent-employee's gross estate of a
lump-sum distribution from a qualified retirement plan to a beneficiary other than the employee's
estate is (are) correct?

1. Lump-sum distributions of payments attributable to the employer's contributions are excluded from
the gross estate.

2. Lump-sum distributions of payments attributable to the decedent-employee's contributions are


excluded from the gross estate.

A. 1 only

B. Both 1 and 2

C. Neither 1 nor 2

D. 2 only

Answer: C

QUESTION NO: 116

All the following statements concerning a federal estate tax deduction for a bequest or gift to a qualified
charity are correct EXCEPT:
A. The amount of a charitable deduction may not exceed 50 percent of a decedent's adjusted gross
estate.

B. An estate may deduct the value of the remainder interest in a charitable remainder trust.

C. A life insurance policy that was assigned to a charity as a gift less than 3 years prior to the insured's
death qualifies for a charitable deduction.

D. The amount of a charitable deduction is reduced by any taxes and administrative expenses
chargeable against the bequest.

Answer: A

QUESTION NO: 117

All the following assets owned by a decedent pass by operation of law or by contract EXCEPT

A. death benefits of a pension plan payable to a named beneficiary

B. jointly owned real estate passing to the surviving joint tenant

C. life insurance payable to a named beneficiary

D. probate assets passing under the terms of a valid will

Answer: D

QUESTION NO: 118

Which of the following statements concerning the estate tax marital deduction is correct?
A. The marital deduction available to a decedent in a community-property state is equal to the total
amount of community property.

B. The marital deduction available to a decedent in a common-law state is limited to a maximum of $1


million.

C. The marital deduction available to a decedent in a common-law state is equal to one half the adjusted
gross estate.

D. The marital deduction available to a decedent in a common-law state is equal to the net amount of
qualifying property passing to the surviving spouse.

Answer: D

QUESTION NO: 119

All the following statements concerning guardians for minors are correct EXCEPT:

A. A guardian has equitable title to the property he administers for the minor.

B. A guardian of the person of a minor may not necessarily be the guardian of the minor's property.

C. A guardian named in a deceased parent's will is not necessarily binding on the court.

D. A special guardian can be appointed by the court to protect a minor's rights in a legal proceeding.

Answer: A

QUESTION NO: 120

Among the assets in a decedent's gross estate is stock in a closely held corporation that was left to a
nephew. The interest passing to the nephew is required to bear the burden of all estate taxes and
expenses. The relevant facts concerning this estate are:
 Adjusted gross estate $1,600,000
 Fair market value of stock in
 the closely held corporation 700,000
 Funeral expenses 30,000
 Executor's commission 50,000
 Federal and state death tax 160,000

What amount of closely held corporate stock can be redeemed under IRC Section 303 so that the
redemption will be treated as a sale or exchange rather than a dividend distribution?

A. $240,000

B. $700,000

C. $ 80,000

D. 0

Answer: A

QUESTION NO: 121

A wealthy individual might consider selling a substantially appreciated property interest in an


installment sale for which of the following reasons?

1. To spread the taxable gain inherent in the property over the period of the installments

2. To provide a buyer who lacks the requisite funds for a lump-sum purchase with the ability to finance
the acquisition

A. 1 only
B. Neither 1 nor 2

C. 2 only

D. Both 1 and 2

Answer: D

QUESTION NO: 122

Requirements for property to qualify for the federal estate tax marital deduction include which of the
following?

1. The property interest must be includible in the decedent's gross estate.

2. The property must pass in such manner that it will be includible in the surviving spouse's estate at
death unless consumed or given away.

A. Both 1 and 2

B. 2 only

C. 1 only

D. Neither 1 nor 2

Answer: A

QUESTION NO: 123


When the owner of a closely held business dies, the payment of a portion of the federal estate tax may
be deferred for a period of several years if the estate otherwise qualifies under the provisions of IRC
Section 6166. Which of the following statements concerning this deferral of federal estate tax is correct?

A. Under certain circumstances, the estate will forfeit its right to tax deferral, and all the remaining
unpaid estate tax will become due and payable immediately.

B. The interest rate on the deferred tax is determined by the prime rate in effect on the date of death.

C. To qualify for the tax deferral, the closely held business must represent more than 50 percent of the
value of the decedent's adjusted gross estate.

D. The interest on the unpaid estate tax is payable over the first 10 years, after which the tax plus
interest on the balance is payable in equal installments for the last 5 years.

Answer: A

QUESTION NO: 124

An executor elects to value the assets of the estate at the alternative valuation date 6 months after
death. Which of the following statements concerning the estate tax value of assets included in this
estate is correct?

A. Property distributed under the will before the alternate valuation date is valued at the date of death.

B. Property that has increased in value since the date of death may be valued at the date of death if the
executor so elects.

C. An annuity included in the gross estate that diminishes with the mere passage of time is includible at
the date of death value.

D. Property sold before the alternate valuation date is valued at the alternate valuation date.

Answer: C
QUESTION NO: 125

A father wants to accumulate funds for his 12-year-old son's college education. On the advice of his
attorney, the father establishes an IRC Section 2503(c) trust and funds it with annual gifts. All the
following statements concerning this arrangement are correct EXCEPT:

A. In the event of the son's death prior to age 21, trust assets must either be payable to the son's estate
or be subject to a general power of appointment held by the son.

B. The trust must be irrevocable.

C. Any accumulated income and all trust principal must be available for distribution to the son when he
attains age 21.

D. The father's annual gift tax exclusion must be reduced by any amount used to pay college tuition
costs.

Answer: D

QUESTION NO: 126

Which of the following acts by a person other than a lawyer is clearly an unauthorized practice of law?

A. A sister drafts a will for her brother using printed forms.

B. A CLU explains to a client how a life insurance policy may solve estate liquidity needs.

C. A CPA designs an estate plan for presentation to a client.

D. A trust officer gives a client advice about the taxation of a trust.

Answer: A
QUESTION NO: 127

A father and son have been farming land owned by the father for the past 12 years. Just prior to his
death, the father was offered $900,000 for his farm because of its possible use as a shopping center. The
son would like to continue to farm the land if it can be included in his father's estate at its current use
value. Additional facts are:

1.Average annual gross rentals from nearby farms of similar acreage are $36,000.

2.Average annual state and local real estate taxes on the farm are $4,000.

3.The interest rate for loans from the Federal Land Bank is 8 percent.

For federal estate tax purposes, the farm method valuation formula would result in a current use value
for the farm of

A. $600,000

B. $300,000

C. $500,000

D. $400,000

Answer: D

QUESTION NO: 128

All the following statements concerning lifetime gifts are correct EXCEPT:
A. A substantial amount of property may be given away over a period of time without the imposition of
the federal gift tax because of the annual exclusion.

B. The amount of gift tax paid within 3 years of death is included in the gross estate.

C. If a wealthy widower lives more than 3 years after making a taxable gift to his sister, the value of the
gift has no effect on his federal estate tax liability.

D. Gifts of life insurance within 3 years of death are included in the donor-insured's gross estate.

Answer: C

QUESTION NO: 129

All the following statements concerning an estate for a term of years are correct EXCEPT:

A. The tenant has the right to possess the property during the term of his interest.

B. An interest may extend beyond the lifetime of the grantor.

C. The tenant may transfer the property at the end of the term of his interest.

D. It is an interest in property established for a specific duration.

Answer: C

QUESTION NO: 130

All the following statements concerning real property ownership by married couples as joint tenants
with right of survivorship are correct EXCEPT:

A. The deceased spouse's interest in the property qualifies for the marital deduction since it passes
outright to the surviving spouse.
B. In common-law states the total value of the property receives a stepped-up tax basis in the estate of
the first spouse to die.

C. Jointly held property between spouses does not pass through the probate estate of the first spouse to
die.

D. All benefits of ownership remain available to the surviving spouse without interruption during the
administration of the deceased spouse's estate.

Answer: B

QUESTION NO: 131

All the following statements concerning qualification of property for the federal estate tax marital
deduction are correct EXCEPT:

A. The property interest passing to the surviving spouse must be includible in the decedent's gross
estate.

B. The property received by a surviving spouse after a successful will contest fails to qualify.

C. The property interest must be includible in the surviving spouse's estate at death unless consumed or
given away.

D. The surviving spouse must receive the property as the beneficial owner rather than as a trustee for
someone else.

Answer: B

QUESTION NO: 132

A widower dies leaving a net probate estate of $300,000. At the time of his death, his descendants are
as follows:
A son, Joe, who has no children;

A deceased daughter, Mary, whose two children, Irene and Sally, survive; and

A daughter, Anne, who has one child, Harry

Assuming that the widower's will provides for the distribution of his assets in equal shares to his
children, per stirpes, which of the following correctly states the amounts each descendant will receive?

A. $60,000 to Joe, $60,000 to Irene, $60,000 to Sally, $60,000 to Anne, and $60,000 to Harry

B. $75,000 to Joe, $75,000 to Irene, $75,000 to Sally, and $75,000 to Anne

C. $100,000 to Joe, $50,000 to Irene, $50,000 to Sally, and $100,000 to Anne

D. $100,000 to Joe, $50,000 to Irene, $50,000 to Sally, $50,000 to Anne, and $50,000 to Harry

Answer: C

QUESTION NO: 133

Which of the following statements concerning guardians is (are) correct?

1. A guardian is appointed by a court.

2. A guardian holds equitable title to the property he administers for his or her ward.

A. Both 1 and 2

B. 2 only
C. Neither 1 nor 2

D. 1 only

Answer: D

QUESTION NO: 134

All the following statements concerning the ownership of real property as joint tenants with right of
survivorship are correct EXCEPT:

A. If the joint tenants are husband and wife, because this is a qualified joint interest, one half the value
of the property will be in the estate of the first spouse to die regardless of which spouse contributed to
the purchase price.

B. If three sisters inherited property as joint tenants with right of survivorship, the entire value of the
property will be in the estate of the first sister to die.

C. If the joint tenants are brother and sister, no portion of the value of the property will be in the sister's
estate if she dies first provided her executor proves that the brother contributed all the funds.

D. If the joint tenants are two brothers and each contributed one half the property's purchase price,
only one half the property's value will be in the estate of the first brother to die if his executor proves
that the other brother contributed half of the purchase price.

Answer: B

QUESTION NO: 135

All the following statements concerning property ownership by a married couple residing in a
community-property state are correct EXCEPT:

A. Income earned by one spouse becomes community property.


B. Property inherited during the marriage is the separate property of the spouse who inherited it.

C. All property that is not separate property is community property.

D. Community property loses its identity when a community-property couple moves to a common-law
state.

Answer: D

QUESTION NO: 136

Which of the following statements concerning the inclusion and valuation of all or part of a commercial
annuity in the estate of an annuitant is correct?

A. A life annuity with a period certain is includible to the extent of the present value of any remaining
guaranteed payments.

B. A joint and survivor annuity is includible to the extent that the survivor paid the purchase price of the
contract.

C. An annuity is includible to the extent of the value determined by the special 10-year averaging rule.

D. If the executor elects the alternate valuation date, an annuity is includible at its replacement cost 6
months after death.

Answer: A

QUESTION NO: 137

Which of the following statements concerning estates and trusts is (are) correct?

1. Both estates and trusts come into being by operation of law.


2. The personal representative of an estate and the trustee have similar fiduciary responsibilities.

A. Both 1 and 2

B. Neither 1 nor 2

C. 1 only

D. 2 only

Answer: D

QUESTION NO: 138

Believing that his death was imminent, a widower gave his son some real estate two years ago and filed
a timely gift tax return. The widower died on January 1st of this year. The additional facts are:

 Widower's basis in the real estate $400,000


 Value of the real estate when gifted 1,000,000
 Value of the real estate on date of death 2,000,000
 Amount of gift tax paid by widower 345,800

Assuming the widower made no additional gifts to his son, all the following statements concerning this
situation are correct EXCEPT:

A. The son's income tax basis in the real estate is $2,000,000.

B. The widower recognized no gain for income tax purposes at the time the gift was made.

C. The gift of the real estate is included in the calculation of the widower's federal estate tax as an
adjusted taxable gift.

D. The gift tax paid is brought back into the widower's gross estate at $345,800.
Answer: A

QUESTION NO: 139

Which of the following statements concerning charitable remainder annuity trusts is correct?

A. The term of charitable remainder annuity arrangements is limited to 10 years.

B. It provides a fixed annuity income interest to a qualified charity.

C. The donor receives an estate tax charitable deduction for the value of the remainder interest.

D. The interest paid to the charity must be paid at least quarterly.

Answer: C

QUESTION NO: 140

To determine whether a taxable gift has been made, the Treasury Regulations require that there must
initially be a definite finding that the

A. donor was a close friend or a relative of the donee

B. property was transferred for less than an adequate and full consideration in money or money's worth

C. transferor's actual state of mind was such that he intended to make a gift

D. property transferred was real property or tangible personal property

Answer: B
QUESTION NO: 141

Important factors in assessing liquidity needs in estate planning include which of the following?

1. The types of assets that comprise the estate

2. The date of drafting the will

A. Neither 1 nor 2

B. 2 only

C. Both 1 and 2

D. 1 only

Answer: D

QUESTION NO: 142

Income earned but unpaid at the time of a decedent's death is deemed to be income in respect of a
decedent (IRD). Which of the following statements concerning IRD is (are) correct?

1. The income is taxable to the person or entity receiving it.

2. The present value of the income is includible in the decedent's gross estate.

A. 1 only

B. 2 only
C. Neither 1 nor 2

D. Both 1 and 2

Answer: D

QUESTION NO: 143

Which of the following types of partial interests in property may be allowed a charitable deduction for
estate tax purposes?

1. A remainder interest in the donor vacation home

2. A testamentary gift of a percentage of a decedent entire interest in property held in trust

A. Both 1 and 2

B. Neither 1 nor 2

C. 1 only

D. 2 only

Answer: C

QUESTION NO: 144

All the following statements concerning real property ownership by married couples as joint tenants
with right of survivorship are correct EXCEPT:
A. In common-law states the total value of the property receives a stepped-up tax basis in the estate of
the first spouse to die.

B. The deceased spouse's interest in the property qualifies for the marital deduction since it passes
outright to the surviving spouse.

C. Jointly held property between spouses does not pass through the probate estate of the first spouse to
die.

D. All benefits of ownership remain available to the surviving spouse without interruption during the
administration of the deceased spouse's estate.

Answer: A

QUESTION NO: 145

Which of the following types of real property ownership will be deemed to be a tenancy in common?

A. Two brothers own equal amounts of all the common stock in a corporation, the only asset of which is
real property.

B. Two brothers are equal partners in a general partnership that owns a piece of real property used in
the partnership business.

C. Two brothers own equal fractional interests in a piece of real property and at the death of one of the
brothers the survivor will own the entire piece of property.

D. Two brothers own equal undivided interests in a piece of real property, with each brother being able
to divest himself of his interest by sale, gift, or will.

Answer: D

QUESTION NO: 146

In all cases a trustee must invest trust assets in accordance with the
A. common law standard of the prudent person

B. stated provisions of the trust instrument

C. requirements of a state's permissive legal list

D. requirements of a state's mandatory legal list

Answer: B

QUESTION NO: 147

The failure of an individual to have a will can result in all the following EXCEPT:

A. Testamentary gifts to charity cannot be made.

B. The decedent's state of domicile might receive the property left by the decedent.

C. Unnecessary death taxes may be imposed.

D. A surviving spouse receives only his or her elective share.

Answer: D

QUESTION NO: 148

All the following statements concerning property ownership by a married couple residing in a
community-property state are correct EXCEPT:

A. Property inherited during the marriage is the separate property of the spouse who inherited it.

B. Income earned by one spouse becomes community property.


C. Community property loses its identity when a community-property couple moves to a common-law
state.

D. All property that is not separate property is community property.

Answer: C

QUESTION NO: 149

An individual who is a resident of State W is also the sole proprietor of a business located in State W. He
owns real property located in State X that is used by the proprietorship. While on vacation in State Y, the
individual meets an untimely death. Under the terms of his will, his entire estate is bequeathed to a
resident of State Z. Which state will tax the real property used by the proprietorship?

A. State W

B. State Z

C. State Y

D. State X

Answer: D

QUESTION NO: 150

All the following items of property will be included in a decedent's gross estate for federal estate tax
purposes EXCEPT

A. the value of property subject to a general power of appointment that the decedent possessed at
death
B. the value of a gratuitous lifetime transfer in which the decedent retained a reversionary interest on
the date of death worth more than 5 percent of the value of the property and which the donee must
survive the decedent to possess

C. the value of all gratuitous lifetime transfers of property made within 3 years of death

D. the value of a gratuitous lifetime transfer in which the decedent retained the right for life to receive
the income from the property

Answer: C

QUESTION NO: 151

To determine whether a taxable gift has been made, the IRS focuses on all the following factors EXCEPT:

A. Was the transferred property real property or personal property?

B. Was the value of the gift property in excess of the annual per-donee exclusion?

C. Did the donor absolutely, irrevocably, and currently divest himself of dominion and control over the
property?

D. Was the property transferred for less than an adequate and full consideration in money or money's
worth?

Answer: A

QUESTION NO: 152

Which of the following is (are) a permissible deduction from a decedent's gross estate to determine the
decedent's adjusted gross estate?

1. Expenses incurred for the benefit of individual heirs.


2. Expenses incurred in the collection of estate assets.

A. Neither 1 nor 2

B. 2 only

C. 1 only

D. Both 1 and 2

Answer: B

QUESTION NO: 153

A father plans to create a trust for the benefit of his 22-year-old son and wishes to take advantage of the
gift tax annual exclusion. He has named a bank as trustee. Which of the following trust provisions would
cause the gifts to be ineligible to qualify for the gift tax annual exclusion?

1. The trust income is to be paid to the son or accumulated at the discretion of the trustee.

2. The income is to be accumulated until the son reaches age 32 when all accumulated income and
principal are to be distributed to him.

A. 2 only

B. 1 only

C. Neither 1 nor 2

D. Both 1 and 2

Answer: D
QUESTION NO: 154

All the following will be brought back into the donor's gross estate for federal estate tax purposes
EXCEPT

A. a gratuitous transfer of real property to a revocable inter vivos trust

B. an outright, gratuitous transfer of real property in contemplation of death

C. the gift taxes paid last year on a gratuitous transfer of real property

D. a gratuitous transfer of real property with a reserved right to use and enjoy it for life

Answer: B

QUESTION NO: 155

All the following will be brought back into the donor's gross estate for federal estate tax purposes
EXCEPT

A. a gratuitous transfer of real property to a revocable inter vivos trust

B. an outright, gratuitous transfer of real property in contemplation of death

C. a gratuitous transfer of real property with a reserved right to use and enjoy it for life

D. the gift taxes paid last year on a gratuitous transfer of real property

Answer: B
QUESTION NO: 156

A person dying without a will loses all the following rights EXCEPT the right to

A. take maximum advantage of the marital deduction

B. have assets pass to heirs

C. give property to a charity

D. name the person to settle the estate

Answer: B

QUESTION NO: 157

Which of the following statements concerning the valuation of intangible personal property in the gross
estate of a decedent is correct?

A. When a minority stockholder in a closely held corporation dies, his stock is valued on the basis of the
"blockage" rule.

B. Certain U.S. Treasury bonds that are used to pay federal estate taxes at par are valued at their market
price on the date of death of the owner.

C. Valuing closely held stock requires the consideration of several factors outlined by IRS rulings.

D. If there were no trades of a listed common stock on the date of the stockholder's death, the stock's
value is based on its average daily price for the previous month prior to the shareholder's death.

Answer: C

QUESTION NO: 158


All the following statements concerning a complex trust are correct EXCEPT:

A. The trustee may make distributions of principal to trust beneficiaries.

B. Beneficiaries must receive all distributable net income in the year received by the trust.

C. Beneficiaries are taxed on their share of distributable net income when received by them.

D. A complex trust may make gifts to charity.

Answer: B

QUESTION NO: 159

All the following statements concerning wills are correct EXCEPT:

A. A testator may lose the capacity to revoke a will prior to death.

B. A codicil is a valid modification of a will.

C. Once a person is named as an executor in a will, he or she is required to serve.

D. In most states a surviving spouse can elect against a will that completely disinherits him or her.

Answer: C

QUESTION NO: 160

Which of the following statements concerning charitable remainder unitrusts is correct?

A. A fixed percentage of not less than 10 percent of the net fair market value of the trust assets is paid
to the noncharitable beneficiaries.
B. The remainder interest is paid to the qualified charity after a term of years not greater than 15 years.

C. The net fair market value of the trust assets are revalued annually.

D. No further contributions may be made to a unitrust after the initial payment.

Answer: C

QUESTION NO: 161

A married man is the sole owner of a small business with an estate tax value of $500,000. In addition, he
and his wife own an office building as joint tenants with right of survivorship which they purchased five
years ago. The building has an estate tax value of $1,500,000. They are considering dissolving the joint
tenancy and retitling the building in the name of the husband as sole owner. All the following
statements concerning this action are correct EXCEPT:

A. At the husband's death, his heirs would get a fully stepped-up tax basis for the property.

B. At the husband's death, it would be easier to qualify his estate for an IRC Section 303 stock
redemption of his business interest.

C. If the husband dies first, the probate costs of his estate could be increased.

D. If the husband dies first and leaves the office building outright to his wife, there would be no federal
estate tax attributed to its inclusion in his gross estate.

Answer: B

QUESTION NO: 162

A wife owns a $100,000 life insurance policy on her husband's life. She has named her son the revocable
beneficiary. Which of the following statements concerning the life insurance is (are) correct?
1. At the husband's death, the interpolated terminal reserve of the policy is a gift to the son.

2. The annual increase in the cash value is a gift to the son.

A. 1 only

B. 2 only

C. Both 1 and 2

D. Neither 1 nor 2

Answer: D

QUESTION NO: 163

Which of the following statements concerning the generation-skipping transfer tax (GSTT) is correct?

A. The GSTT rate applicable to a transfer depends upon the amount of the gift.

B. The lifetime exemption shelters a maximum of $1 million of transfers to grandchildren from GSTT for
the current year.

C. Tuition payments made by a grandparent directly to a university for a grandchild's education are
exempt from GSTT.

D. The annual exclusion against GSTT shelters gifts by a grandparent to a trust benefitting multiple
grandchildren.

Answer: C

QUESTION NO: 164


Which of the following statements concerning pooled-income funds is (are) correct?

1. The fund contains commingled donations from many sources.

2. A decedent donation purchases units in the fund which generate income that is paid at least annually
to a charity.

A. 2 only

B. Neither 1 nor 2

C. Both 1 and 2

D. 1 only

Answer: D

QUESTION NO: 165

Which of the following statements concerning ownership of property under a tenancy by the entirety is
correct?

A. It is a form of property ownership that applies only to personal property.

B. The property will be in the probate estate of the first joint tenant to die.

C. One tenant can freely transfer his or her property interest to a third person.

D. It is a form of property ownership available only to married persons.

Answer: D
QUESTION NO: 166

Among the assets in a decedent's gross estate is stock in a closely held corporation that was left to a
nephew. The interest passing to the nephew is required to bear the burden of all estate taxes and
expenses. The relevant facts concerning this estate are:

 Adjusted gross estate $1,600,000


 Fair market value of stock in the
 closely held corporation 700,000
 Funeral expenses 30,000
 Executor's commission 50,000
 Federal and state death tax 160,000

What amount of closely held corporate stock can be redeemed under IRC Section 303 so that the
redemption will be treated as a sale or exchange rather than a dividend distribution?

A. 0

B. $ 80,000

C. $700,000

D. $240,000

Answer: D

QUESTION NO: 167

A man died in February of this year. Last year, when he learned that he had a terminal illness, he
immediately made the following gifts and filed the required gift tax return:

Fair Market Value

Gift of listed stock to a


 qualified charity $100,000
 Gift of listed bonds to his wife 200,000
 Gift of a boat to his son 10,000
 Gift of a sports car to his daughter 10,000

What amount must be brought back to the man's estate as an adjusted taxable gift in the calculation of
his federal estate taxes?

A. $320,000

B. 0

C. $280,000

D. $ 90,000

Answer: B

QUESTION NO: 168

Which of the following statements concerning the inclusion and valuation of all or part of a commercial
annuity in the estate of an annuitant is (are) correct?

1. A life annuity with a period certain is includible to the extent of the present value of any remaining
guaranteed payments.

2. If the executor elects the alternate valuation date, an annuity is includible at its replacement cost 6
months after death.

A. 2 only

B. Both 1 and 2
C. Neither 1 nor 2

D. 1 only

Answer: D

QUESTION NO: 169

On January 1, 2004 a father gave his daughter a $50,000 straight (ordinary) life insurance policy on his
life. Premiums are paid annually. The pertinent facts about the policy are:

Date of issue: July 1, 1992

Premium paid on July 1, 2003 $800

Terminal reserve on July 1, 2003 5,000

Terminal reserve on July 1, 2004 6,000

What is the value of the policy for federal gift tax purposes?

A. $5,400

B. $5,800

C. $5,900

D. $50,000

Answer: C
QUESTION NO: 170

An executor elects to value the assets of the estate at the alternative valuation date 6 months after
death. Which of the following statements concerning the estate tax value of assets included in this
estate is correct?

A. Property distributed under the will before the alternate valuation date is valued at the date of death.

B. An annuity included in the gross estate that diminishes with the mere passage of time is includible at
the date of death value.

C. Property sold before the alternate valuation date is valued at the alternate valuation date.

D. Property that has increased in value since the date of death may be valued at the date of death if the
executor so elects.

Answer: B

QUESTION NO: 171

All the following are grounds for contesting a will EXCEPT:

A. The testator did not have testamentary capacity.

B. The widow was bequeathed less than her intestate share.

C. The instrument is a forgery.

D. The testator executed a later valid will.

Answer: B

QUESTION NO: 172


Which of the following statements concerning antenuptial agreements is (are) correct?

1. They are frequently used to protect the interests of children of former marriages.

2. They can legally deprive a surviving spouse of his or her elective share of a deceased spouse's estate.

A. Both 1 and 2

B. 1 only

C. 2 only

D. Neither 1 nor 2

Answer: A

QUESTION NO: 173

Which of the following types of real property ownership will be deemed to be a tenancy in common?

A. Two brothers own equal amounts of all the common stock in a corporation, the only asset of which is
real property.

B. Two brothers own equal undivided interests in a piece of real property, with each brother being able
to divest himself of his interest by sale, gift, or will.

C. Two brothers own equal fractional interests in a piece of real property and at the death of one of the
brothers the survivor will own the entire piece of property.

D. Two brothers are equal partners in a general partnership that owns a piece of real property used in
the partnership business.

Answer: B
QUESTION NO: 174

Which of the following statements concerning estates and trusts is (are) correct?1. Both estates and
trusts come into being by operation of law. 2. The personal representative of an estate and the trustee
have similar fiduciary responsibilities.

A. Both 1 and 2

B. 1 only

C. Neither 1 nor 2

D. 2 only

Answer: D

QUESTION NO: 175

A wife makes outright gifts of $76,000 to her son this year, and her husband agrees to split the gifts with
her. Which of the following correctly states the amount of the taxable gifts?

A. Wife $27,000, husband $27,000

B. Wife $37,000, husband $37,000

C. Wife $17,000, husband $37,000

D. Wife $54,000, husband 0

Answer: A
QUESTION NO: 176

The following are facts concerning a decedent's estate:

 Taxable estate $1,700,000


 Pre-1977 taxable gifts 200,000
 Post-1976 adjusted taxable gifts 50,000
 Post-1976 gifts made to a qualified charity 100,000

The tentative tax base of this estate is

A. $1,750,000

B. $1,850,000

C. $1,900,000

D. $1,700,000

Answer: A

QUESTION NO: 177

Which of the following statements concerning the obligation of the personal representative of a
decedent to file a federal estate tax return is (are) correct?

1. A return must be filed by a decedent's estate if the gross estate plus adjusted taxable gifts equals the
applicable exclusion amount or more, even though no federal estate tax is due.

2. Unless extensions are granted, the return must be filed and the tax paid within 9 months of the
decedent's death.
A. Neither 1 nor 2

B. Both 1 and 2

C. 1 only

D. 2 only

Answer: B

QUESTION NO: 178

Mr. Barlow died early this year. Under the terms of his will he left all his real estate and tangible
personal property to his son. All the remainder of his probate estate was left to his wife, Mrs. Barlow.
The following is a list of Mr. Barlow's probate assets and their fair market values at the time of his death:

 Commercial real estate $200,000


 Furniture and fixtures 100,000
 Listed common stock 150,000
 Notes receivable 250,000

In addition, Mrs. Barlow owned a $400,000 life insurance policy on Mr. Barlow's life with Mr. Barlow's
estate designated as beneficiary. Based on this information, what is the amount of property in Mr.
Barlow's estate qualifying for the federal estate tax marital deduction?

A. $800,000

B. $400,000

C. $1,100,000

D. $150,000

Answer: A
QUESTION NO: 179

All the following powers held by the grantor of an irrevocable trust will cause the trust assets to be
brought back into the estate of the grantor EXCEPT the power to

A. terminate the trust

B. designate who shall enjoy the trust income

C. add principal to the trust

D. change the trust remainderpersons

Answer: C

QUESTION NO: 180

Which of the following provisions is (are) generally common to all buy-sell agreements?

1. Provisions specifying how the purchase price is to be funded.

2. Statement indicating the purpose of the agreement.

A. 2 only

B. Neither 1 nor 2

C. 1 only

D. Both 1 and 2
Answer: D

QUESTION NO: 181

All the following items are allowed as a deduction from a decedent's gross estate to determine the
decedent's adjusted gross estate EXCEPT:

A. Payments of estate debts.

B. Expenses incurred in the presentation of probate assets.

C. Costs of distributing probate assets to estate beneficiaries.

D. Expenses incurred for the benefit of individual heirs.

Answer: D

QUESTION NO: 182

An individual who is a resident of State W is also the sole proprietor of a business located in State W. He
owns real property located in State X that is used by the proprietorship. While on vacation in State Y, the
individual meets an untimely death. Under the terms of his will, his entire estate is bequeathed to a
resident of State Z. Which state will tax the real property used by the proprietorship?

A. State X

B. State W

C. State Z

D. State Y

Answer: A
QUESTION NO: 183

All the following statements concerning a complex trust are correct EXCEPT:

A. Beneficiaries must receive all distributable net income in the year received by the trust.

B. The trustee may make distributions of principal to trust beneficiaries.

C. A complex trust may make gifts to charity.

D. Beneficiaries are taxed on their share of distributable net income when received by them.

Answer: A

QUESTION NO: 184

Which of the following statements concerning pooled-income funds is (are) correct?

1. A pooled income fund is similar to a mutual fund maintained by a qualified charity.

2. It is an irrevocable arrangement in which the remainder interest passes to charity.

A. Neither 1 nor 2

B. 1 only

C. Both 1 and 2

D. 2 only
Answer: C

QUESTION NO: 185

The primary objective in estate planning is to

A. preserve the marital deduction and applicable credit amount credit

B. reflect accurately the client's wishes concerning the disposition of his or her wealth

C. reduce estate taxes to the lowest amount possible

D. prevent the intestate distribution of assets

Answer: B

QUESTION NO: 186

Which of the following statements concerning the generation-skipping transfer tax (GSTT) is correct?

A. The applicable credit amount may be applied against generation-skipping transfers.

B. An annual exclusion is available for direct skip gifts to a grandchild.

C. The GSTT is inapplicable to transfers to nonrelated transferees.

D. The GSTT is imposed only if no federal estate or gift tax applies to a transfer.

Answer: B
QUESTION NO: 187

Which of the following statements concerning both the joint tenancy with right of survivorship and the
tenancy by the entirety forms of real property ownership is correct?

A. Both forms of ownership are restricted to two equal owners at any one time.

B. Both forms of ownership provide that an owner can sell his interest in the property at any time
without destroying the form of ownership.

C. Both forms of ownership provide that a deceased owner's interest passes to the surviving owner.

D. Both forms of ownership are restricted to husband and wife.

Answer: C

QUESTION NO: 188

All the following statements concerning an estate for a term of years are correct EXCEPT:

A. An interest may extend beyond the lifetime of the grantor.

B. The tenant may transfer the property at the end of the term of his interest.

C. The tenant has the right to possess the property during the term of his interest.

D. It is an interest in property established for a specific duration.

Answer: B

QUESTION NO: 189

A single man with substantial assets and income is supporting his 80-year-old partially senile mother
with monthly cash gifts. He is trying to find a practical way to support his mother while at the same time
saving federal gift and income taxes without giving up ultimate control of any assets. Which of the
following courses of action will best accomplish these objectives?

A. Make her a gift of enough corporate bonds from his portfolio so that she will be able to support
herself from the interest payments

B. Make her an interest free loan with a principal amount large enough to produce sufficient income for
her support when invested in corporate bonds

C. Make her annual gifts of enough interest income from the tax free municipal bonds in his portfolio so
that she will be able to support herself

D. Purchase corporate bonds that pay interest in an amount sufficient for her to support herself and
assign the interest payments to her

Answer: C

QUESTION NO: 190

A father died leaving his property equally to his wealthy son and his poor daughter. The son wishes to
disclaim his share of the inheritance so that it will pass to his sister without his incurring any gift tax
liability. In this situation, all the following acts on the part of the son are required EXCEPT:

A. His refusal to accept the inheritance must be received by the executor of his father's estate within 9
months of his father's death.

B. He must not have received any part of his inheritance or any income from it prior to his refusal to
accept it.

C. His refusal to accept the inheritance must direct specifically that his sister is to receive it instead.

D. His refusal to accept the inheritance must be in writing.

Answer: C
QUESTION NO: 191

Which of the following factors is (are) used to make a choice between having an entity-purchase or
cross-purchase partnership buy-sell agreement?

1. The cost basis of the partners' business interests.

2. The amount of the partners?individual personal net worths.

A. 1 only

B. Neither 1 nor 2

C. 2 only

D. Both 1 and 2

Answer: A

QUESTION NO: 192

Generally the courts will accept as the federal estate tax value of a closely held corporate business the
price established by a buy-sell agreement if all the following conditions are met EXCEPT:

A. The agreement requires a deceased shareholder's executor to sell the stock at the price specified in
the agreement.

B. The agreement as to per-share value is fair, adequate, and made at arm's length.

C. The agreement requires the payment of liquidated damages to the survivors if the executor fails to
carry out its terms.
D. The agreement requires a shareholder to first offer his stock to the corporation or other shareholders
at the specified price if he wishes to sell it during his lifetime.

Answer: C

QUESTION NO: 193

Which of the following statements concerning the obligation of the personal representative of a
decedent to file a federal estate tax return is (are) correct?

1. A return must be filed by a decedent's estate if the gross estate plus adjusted taxable gifts equals the
applicable exclusion amount or more, even though no federal estate tax is due.

2. Unless extensions are granted, the return must be filed and the tax paid within 9 months of the
decedent's death.

A. 1 only

B. Neither 1 nor 2

C. Both 1 and 2

D. 2 only

Answer: C

QUESTION NO: 194

A widower dies leaving a net probate estate of $300,000. At the time of his death, his descendants are
as follows:
 A son, Joe, who has no children;
 A deceased daughter, Mary, whose two children, Irene and Sally, survive; and
 A daughter, Anne, who has one child, Harry

Assuming that the widower's will provides for the distribution of his assets in equal shares to his
children, per stirpes, which of the following correctly states the amounts each descendant will receive?

A. $100,000 to Joe, $50,000 to Irene, $50,000 to Sally, $50,000 to Anne, and $50,000 to Harry

B. $100,000 to Joe, $50,000 to Irene, $50,000 to Sally, and $100,000 to Anne

C. $60,000 to Joe, $60,000 to Irene, $60,000 to Sally, $60,000 to Anne, and $60,000 to Harry

D. $75,000 to Joe, $75,000 to Irene, $75,000 to Sally, and $75,000 to Anne

Answer: B

QUESTION NO: 195

An executor may value assets as of the date of death or the alternate valuation date 6 months after
death. Assuming the executor elects the alternate valuation date, all the following statements are
correct EXCEPT:

A. Property distributed under the will within the alternate valuation period is valued at the date of
death.

B. Property sold by the executor before the alternate valuation date is valued at its sale price.

C. A property interest that diminishes with the mere passage of time, such as a patent, is includible at
the date of death value.

D. Property that has increased in value since the date of death is valued at the alternate valuation date.
Answer: A

QUESTION NO: 196

Which of the following transactions is a taxable gift for federal gift tax purposes?

A. A mother purchased listed common stocks titling them in joint names with her daughter and the
stocks have not yet been sold.

B. A grandmother purchased a U.S. savings bond that is registered as payable to her and her two
grandchildren and the bond has not yet been surrendered for cash.

C. A man deeded real estate to his sister but did not record the deed nor did he deliver the deed to his
sister.

D. A father made a deposit of $100,000 into a bank account titled jointly with his son, and the son has
not yet made any withdrawals.

Answer: A

QUESTION NO: 197

Many trust instruments provide for the removal of the original trustee. Valid reasons for removing the
original trustee include which of the following?

1. A shift in trust situs is desirable because of changes in law.

2. The beneficiary has moved his or her residence to a distant state.

A. Neither 1 nor 2
B. 1 only

C. 2 only

D. Both 1 and 2

Answer: D

QUESTION NO: 198

A man recently died with only probate assets. Under the terms of his will, he left his entire probate
estate outright to his wife. The following are relevant facts about the estate:

 Gross estate $1,700,000


 Estate administration expenses 30,000
 Debts of decedent 65,000
 Allowable funeral expenses 5,000

The amount of the allowable marital deduction is

A. $1,635,000

B. $1,695,000

C. $1,665,000

D. $1,600,000

Answer: D

QUESTION NO: 199


When the owner of a closely held business dies, the payment of a portion of the federal estate tax may
be deferred for a period of several years if the estate otherwise qualifies under the provisions of IRC
Section 6166. Which of the following statements concerning this deferral of federal estate tax is correct?

A. To qualify for the tax deferral, the closely held business must represent more than 50 percent of the
value of the decedent's adjusted gross estate.

B. Under certain circumstances, the estate will forfeit its right to tax deferral, and all the remaining
unpaid estate tax will become due and payable immediately.

C. The interest on the unpaid estate tax is payable over the first 10 years, after which the tax plus
interest on the balance is payable in equal installments for the last 5 years.

D. The interest rate on the deferred tax is determined by the prime rate in effect on the date of death.

Answer: B

QUESTION NO: 200

Which of the following statements concerning both the joint tenancy with right of survivorship and the
tenancy by the entirety forms of real property ownership is correct?

A. Both forms of ownership are restricted to two equal owners at any one time.

B. Both forms of ownership provide that an owner can sell his interest in the property at any time
without destroying the form of ownership.

C. Both forms of ownership provide that a deceased owner's interest passes to the surviving owner.

D. Both forms of ownership are restricted to husband and wife.

Answer: C
QUESTION NO: 201

A man recently died with only probate assets. Under the terms of his will, he left his entire probate
estate outright to his wife. The following are relevant facts about the estate:

 Gross estate $2,000,000


 Estate administration expenses 50,000
 Debts of decedent 200,000
 Allowable funeral expenses 5,000

The amount of the allowable marital deduction is

A. $1,750,000

B. $1,745,000

C. $1,795,000

D. $1,800,000

Answer: B

QUESTION NO: 202

Which of the following factors is (are) used to make a choice between having an entity-purchase or
cross-purchase partnership buy-sell agreement?

1. The cost basis of the partners' business interests.

2. The amount of the partners?individual personal net worths.


A. 2 only

B. 1 only

C. Both 1 and 2

D. Neither 1 nor 2

Answer: B

QUESTION NO: 203

All the following will be brought back into the donor's gross estate for federal estate tax purposes
EXCEPT

A. a gratuitous transfer of real property to a revocable inter vivos trust

B. a gratuitous transfer of real property with a reserved right to use and enjoy it for life

C. the gift taxes paid last year on a gratuitous transfer of real property

D. an outright, gratuitous transfer of real property in contemplation of death

Answer: D

QUESTION NO: 204

Which of the following statements concerning property is correct?

A. A tree growing on land is tangible personal property.

B. A mortgage on real estate is real property.

C. Any property that is not real property is personal property.


D. A bond issue secured solely by a corporation assets is tangible personal property.

Answer: C

QUESTION NO: 205

Nontax benefits of lifetime gifts include all the following EXCEPT to

A. reduce probate and administrative costs

B. avoid claims of creditors evidenced by liens

C. provide for the education of the donee

D. obtain privacy that is not possible to obtain through testamentary transfers

Answer: B

QUESTION NO: 206

All the following statements concerning a grantor-retained unitrust (GRUT) are correct EXCEPT:

A. When the grantor dies during the retained interest term, the estate tax benefits are increased.

B. When the trust is created, the grantor makes an irrevocable transfer to remainderperson(s).

C. The transfer of the remainder interest in the trust is a current gift for gift tax purposes.

D. The grantor's applicable credit amount may be applied to the gifted remainder interest in the trust.

Answer: A
QUESTION NO: 207

A wife makes outright gifts of $66,000 this year to her son, and her husband agrees to split the gifts with
her. Which of the following correctly states the amount of the taxable gifts?

A. Wife $12,000, husband $32,000

B. Wife $32,000, husband $32,000

C. Wife $22,000, husband $22,000

D. Wife $44,000, husband 0

Answer: C

QUESTION NO: 208

If a grantor establishes an irrevocable trust, the income of the trust will be taxed to the grantor if it is
used to pay premiums for life insurance on the life of

A. the spouse of the grantor

B. the father of the grantor

C. a child of the grantor

D. a grandchild of the grantor

Answer: A
QUESTION NO: 209

Which of the following statements concerning pooled-income funds is (are) correct?

1. A pooled income fund is similar to a mutual fund maintained by a qualified charity.

2. It is an irrevocable arrangement in which the remainder interest passes to charity.

A. Both 1 and 2

B. 2 only

C. Neither 1 nor 2

D. 1 only

Answer: A

QUESTION NO: 210

Which of the following areas of consideration present common ethical issues for the estate planner?

A. Consistency

B. Compatibility

C. Compensation

D. Contracts

Answer: C
QUESTION NO: 211

A married man died intestate. In addition to his wife, he was survived by two minor children and both
his parents. Which of the following statements describes the typical intestate distribution in this
situation?

A. The widow receives at least one-third of the estate and the children divide the remainder of the
estate equally.

B. The widow receives one-third of the estate and the remainder is divided equally among the two
children and the parents of the decedent.

C. The widow receives half the estate and the remaining half is divided equally between the decedent's
parents.

D. The widow receives the entire probate estate as trustee for the benefit of the two minor children.

Answer: A

QUESTION NO: 212

Having a will enables an individual to do which of the following?

1. Make testamentary bequests to selected charitable organizations

2. Make specific bequests to selected friends

A. 1 only

B. Neither 1 nor 2

C. 2 only
D. Both 1 and 2

Answer: D

QUESTION NO: 213

Which of the following statements concerning a testamentary trust is correct?

A. It saves federal and state death taxes at the death of the grantor.

B. The assets in the trust are free of probate costs.

C. It becomes effective only at the death of the grantor.

D. The trust terms must be included in the will.

Answer: C

QUESTION NO: 214

Mr. Allen died early this year survived by his spouse Mrs. Allen. Among the items of family property are:

1.A $200,000 life insurance policy on Mr. Allen's life with Mrs. Allen designated as beneficiary. Mrs. Allen
has been the owner of the policy ever since it was issued 4 years ago.

2.The family residence with a fair market value of $300,000. Mr. and Mrs. Allen own the residence
jointly with the right of survivorship even though Mr. Allen purchased it with his separate funds.

3.A $20,000 bank account. Mr. and Mrs. Allen own the account jointly with the right of survivorship
even though Mrs. Allen made all the deposits.
What amount of the family property will be included in Mr. Allen's gross estate for federal estate tax
purposes?

A. $350,000

B. $360,000

C. $300,000

D. $160,000

Answer: D

QUESTION NO: 215

Mr. Allen died early this year survived by his spouse Mrs. Allen. Among the items of family property are:

1.A $200,000 life insurance policy on Mr. Allen's life with Mrs. Allen designated as beneficiary. Mrs. Allen
has been the owner of the policy ever since it was issued 4 years ago.

2.The family residence with a fair market value of $300,000. Mr. and Mrs. Allen own the residence
jointly with the right of survivorship even though Mr. Allen purchased it with his separate funds.

3.A $20,000 bank account. Mr. and Mrs. Allen own the account jointly with the right of survivorship
even though Mrs. Allen made all the deposits.

What amount of the family property will be included in Mr. Allen's gross estate for federal estate tax
purposes?
A. $360,000

B. $300,000

C. $160,000

D. $350,000

Answer: C

QUESTION NO: 216

Which of the following statements concerning the ownership of real property as joint tenants with right
of survivorship is correct?

A. If the joint tenants are husband and wife, because this is a qualified joint interest, the entire value of
the property will be in the estate of the husband regardless of which spouse dies first.

B. If the joint tenants are brother and sister and the brother contributed all the funds to purchase the
property, one-half of the value of the property will be excluded from the sister's estate if she dies first.

C. If three sisters inherited property as joint tenants with right of survivorship, the entire value of the
property will be in the estate of the first to die.

D. If the joint tenants are two brothers and each contributed one half the property's purchase price,
only one half the property's value will be in the estate of the first brother to die if his executor proves
that the other brother contributed half of the purchase price.

Answer: D

QUESTION NO: 217

All the following will be brought back into the donor's gross estate for federal estate tax purposes
EXCEPT
A. the gift taxes paid last year on a gratuitous transfer of real property

B. a gratuitous transfer of real property with a reserved right to use and enjoy it for life

C. an outright, gratuitous transfer of real property in contemplation of death

D. a gratuitous transfer of real property to a revocable inter vivos trust

Answer: C

QUESTION NO: 218

All the following statements concerning the ownership of real property as joint tenants with right of
survivorship are correct EXCEPT:

A. If the joint tenants are two brothers and each contributed one half the property's purchase price, only
one half the property's value will be in the estate of the first brother to die if his executor proves that
the other brother contributed half of the purchase price.

B. If the joint tenants are husband and wife, because this is a qualified joint interest, one half the value
of the property will be in the estate of the first spouse to die regardless of which spouse contributed to
the purchase price.

C. If the joint tenants are brother and sister, no portion of the value of the property will be in the sister's
estate if she dies first provided her executor proves that the brother contributed all the funds.

D. If three sisters inherited property as joint tenants with right of survivorship, the entire value of the
property will be in the estate of the first sister to die.

Answer: D

QUESTION NO: 219


Which of the following statements concerning filing the federal estate tax return is correct?

A. A one-year extension for filing the estate tax return is granted when the estate contains a closely held
business interest.

B. An automatic two-year extension for filing the estate tax return is granted when the decedent dies
overseas.

C. The estate tax return must be filed within 9 months of death unless an extension is granted by the
IRS.

D. For persons dying under current law, an estate tax return must be filed for all U.S. citizen decedents.

Answer: C

QUESTION NO: 220

All the following statements concerning an entity-purchase buy-sell agreement for a partnership are
correct EXCEPT:

A. Both the partners and the partnership are parties to the agreement that provides for business
continuation.

B. The partnership makes payments to the decedent-partner's estate to liquidate the partnership
interest held by the estate.

C. The partnership, in effect, liquidates the interest held by the decedent-partner's estate.

D. It is the surviving partners who purchase the decedent's business interest so that the partnership can
afford to pay the decedent's estate.

Answer: D
QUESTION NO: 221

All the following are conditions that must be met if an otherwise nonqualified terminable interest is to
qualify (as QTIP) for the federal estate tax marital deduction EXCEPT:

A. The surviving spouse must make a qualified disclaimer to all other property in the deceased spouse's
estate within 9 months of death.

B. No person can be given the right to direct that the property go to anyone other than the surviving
spouse as long as the surviving spouse is alive.

C. The surviving spouse must be given a lifetime right to receive all the property's income at least
annually.

D. The deceased spouse's executor must make an irrevocable election to have the property includible in
the surviving spouse's gross estate.

Answer: A

QUESTION NO: 222

Among the assets in a decedent's gross estate is stock in a closely held corporation that was left to a
nephew. The interest passing to the nephew is required to bear the burden of all estate taxes and
expenses. The relevant facts concerning this estate are:

 Adjusted gross estate $1,600,000


 Fair market value of stock in the
 closely held corporation 700,000
 Funeral expenses 30,000
 Executor's commission 50,000
 Federal and state death tax 160,000

What amount of closely held corporate stock can be redeemed under IRC Section 303 so that the
redemption will be treated as a sale or exchange rather than a dividend distribution?
A. $240,000

B. 0

C. $700,000

D. $ 80,000

Answer: A

QUESTION NO: 223

All the following statements concerning the generation-skipping transfer tax (GSTT) are correct EXCEPT:

A. Each individual has an aggregate $1.5 million exemption against GSTT.

B. Tuition payments made by a grandparent directly to a university for a grandchild's education are
exempt from GSTT.

C. The liability for GSTT falls upon the donee regardless of the type of transfer.

D. Direct skip gifts by a grandparent of up to $11,000 can be made to each grandchild without GSTT
liability due to an annual exclusion.

Answer: C

QUESTION NO: 224

A father deeded a house as a gift to his daughter in 1990 but retained the right to live in it until his
death. He died this year while still living in the house. The following are relevant facts: The father bought
the property in 1980 for $130,000. The fair market value of the property when the gift was made in
1990 was $150,000. The father filed a timely gift tax return but paid no gift tax because of the
applicable credit amount. The fair market value of the property at the father's death was $220,000. The
daughter sold the property 3 months after her father's death for $220,000. She had a gain of

A. 0

B. $140,000

C. $220,000

D. $120,000

Answer: A

QUESTION NO: 225

All the following statements concerning ownership of property in the form of a joint tenancy with right
of survivorship are correct EXCEPT:

A. Joint tenants need not be related either by blood or marriage.

B. Either real property or personal property may be the subject of this type of ownership.

C. Upon the death of a joint tenant, his interest in the property passes to his estate or heirs.

D. All joint tenants must have equal interests in the property.

Answer: C

QUESTION NO: 226

A father deeded a house as a gift to his daughter in 1990 but retained the right to live in it until his
death. He died this year, while still living in the house. The following are relevant facts:
The father bought the property in 1980 for $140,000. The fair market value of the property when the
gift was made in 1990 was $170,000. The father filed a timely gift tax return but paid no gift tax because
of the applicable credit amount. The fair market value of the property at the father's death was
$200,000. The daughter sold the property 3 months after her father's death for $200,000. She had a
gain of

A. $200,000

B. $160,000

C. $130,000

D. 0

Answer: D

QUESTION NO: 227

A single man with substantial assets and income is supporting his 80-year-old partially senile mother
with monthly cash gifts. He is trying to find a practical way to support his mother while at the same time
saving federal gift and income taxes without giving up ultimate control of any assets. Which of the
following courses of action will best accomplish these objectives?

A. Purchase corporate bonds that pay interest in an amount sufficient for her to support herself and
assign the interest payments to her

B. Make her a gift of enough corporate bonds from his portfolio so that she will be able to support
herself from the interest payments

C. Make her annual gifts of enough interest income from the tax free municipal bonds in his portfolio so
that she will be able to support herself

D. Make her an interest free loan with a principal amount large enough to produce sufficient income for
her support when invested in corporate bonds

Answer: C
QUESTION NO: 228

The owner of a successful business wishes to sell it to his employee-son so that he can retire. The
business is worth substantially more than the owner's basis. The owner and the employee-son have
agreed to an installment sale. Which of the following statements concerning this sale is (are) correct?

1. The present value of any unpaid installments remaining at the owner's death is includible in his
estate.

2. Installment payments are received free of income tax until the seller recovers his basis.

A. Neither 1 nor 2

B. Both 1 and 2

C. 1 only

D. 2 only

Answer: C

QUESTION NO: 229

All the following are proper actions on the part of a trustee EXCEPT

A. purchasing assets for personal use from the trust at their fair market value with the approval of all
beneficiaries

B. investing trust assets in speculative securities in accordance with the provisions of the trust
instrument
C. placing cash from the sale of securities in a non-interest bearing checking account for an extended
period of time

D. purchasing securities in good faith from a third party just prior to a sharp decrease in their value

Answer: C

QUESTION NO: 230

Which of the following statements concerning antenuptial agreements is (are) correct?

1. They are frequently used prior to second marriages.

2. They can affect a surviving spouse's right to receive an intestate share of a deceased spouse's estate.

A. Neither 1 nor 2

B. 2 only

C. 1 only

D. Both 1 and 2

Answer: D

QUESTION NO: 231

Requirements for property to qualify for the federal estate tax marital deduction include which of the
following?
1. The property interest must be includible in the decedent's gross estate.

2. The property must pass in such manner that it will be includible in the surviving spouse's estate at
death unless consumed or given away.

A. 1 only

B. Both 1 and 2

C. Neither 1 nor 2

D. 2 only

Answer: B

QUESTION NO: 232

Among the assets in a decedent's gross estate is stock in a closely held corporation that was left to a
nephew. The interest passing to the nephew is required to bear the burden of all estate taxes and
expenses. The relevant facts about this estate are:

 Adjusted gross estate $1,200,000


 Fair market value of stock in the
 closely held corporation 500,000
 Administration and funeral expenses 25,000
 State inheritance taxes 40,000
 Federal estate taxes 160,000

What amount of closely held corporate stock may be redeemed under IRC Section 303 so that the
redemption will be treated as a sale or exchange rather than a dividend distribution?

A. $225,000
B. $500,000

C. 0

D. $ 65,000

Answer: A

QUESTION NO: 233

All the following statements concerning transfers at death under a will are correct EXCEPT:

A. Specific bequests of a decedent's property are satisfied prior to distribution of the decedent's
residuary estate.

B. If during lifetime a decedent disposed of property that was the subject of a specific bequest,
ademption occurs.

C. It is common for a will to contain a clause that exempts the executor from posting bond.

D. The most appropriate way to sever a joint tenancy with right of survivorship is for the joint tenant-
decedent to make a specific bequest of the property under a will.

Answer: D

QUESTION NO: 234

The personal representative of a decedent has the duty to file which of the following income tax
returns?

1. The decedent's final income tax return


2. The estate's income tax return

A. 2 only

B. 1 only

C. Both 1 and 2

D. Neither 1 nor 2

Answer: C

QUESTION NO: 235

Believing that his death was imminent, a widower gave his son some real estate two years ago, and filed
a timely gift tax return. The widower died on January 1st of this year. Additional facts are:

 Widower's basis in the real estate $200,000


 Value of real estate when gifted 510,000
 Value of real estate on date of death 1,000,000
 Amount of gift tax paid by widower 159,500

Assuming the widower made no additional gifts to his son, all the following statements concerning this
situation are correct EXCEPT:

A. The gift of the real estate is included in the calculation of the widower's federal estate tax as an
adjusted taxable gift.

B. The gift tax paid is brought back into the widower's gross estate at $159,500.

C. The widower recognized no capital gain for income tax purposes at the time the gift was made.

D. The son's income tax basis in the real estate is $1,000,000.


Answer: D

QUESTION NO: 236

Which of the following statements concerning ownership of property under a tenancy by the entirety is
correct?

A. It is a form of property ownership that applies only to personal property.

B. The property will be in the probate estate of the first joint tenant to die.

C. It is a form of property ownership available only to married persons.

D. One tenant can freely transfer his or her property interest to a third person.

Answer: C

QUESTION NO: 237

A man established and funded an irrevocable trust and named a bank as trustee. All income from the
trust is to be paid to his four grandchildren. Which of the following powers retained by the grantor of
the trust will cause all or a portion of the trust assets to be includible in his gross estate for federal
estate tax purposes?

1. The power to add principal to the trust

2. The power to vary the amounts of trust income paid to each grandchild

A. Neither 1 nor 2

B. 1 only
C. 2 only

D. Both 1 and 2

Answer: C

QUESTION NO: 238

Which of the following life insurance settlement options will qualify for the federal estate tax marital
deduction?

1. Proceeds left to the surviving spouse under the interest option, with interest payable to the surviving
spouse who has the unrestricted right to withdraw proceeds and with any proceeds not withdrawn
payable equally to her children per stirpes

2. Proceeds left to the surviving spouse under an installment option, with any installments remaining at
her death to be commuted and paid to her estate

A. Both 1 and 2

B. 2 only

C. Neither 1 nor 2

D. 1 only

Answer: A

QUESTION NO: 239


All the following statements concerning a grantor-retained annuity trust (GRAT) or a grantor-retained
unitrust (GRUT) are correct EXCEPT:

A. A grantor receives an annual payout from a GRUT that is based on the value that the trust grows or
contracts to each year.

B. After the initial transfer to a GRAT, the grantor is prohibited from making additional contributions to
the trust.

C. For gift tax purposes a grantor's retained interest in these trusts is valued at zero.

D. Post-transfer appreciation of property transferred to these trusts is excluded from the grantor's gross
estate.

Answer: C

QUESTION NO: 240

Which of the following types of real property ownership will be deemed to be a tenancy in common?

A. Two brothers are equal partners in a general partnership that owns a piece of real property used in
the partnership business.

B. Two brothers own equal amounts of all the common stock in a corporation, the only asset of which is
real property.

C. Two brothers own equal fractional interests in a piece of real property and at the death of one of the
brothers the survivor will own the entire piece of property.

D. Two brothers own equal undivided interests in a piece of real property, with each brother being able
to divest himself of his interest by sale, gift, or will.

Answer: D
QUESTION NO: 241

When the owner of a closely held business dies, the payment of a portion of the federal estate tax may
be deferred for a period of several years if the estate otherwise qualifies under the provisions of IRC
Section 6166. Which of the following statements concerning this deferral of federal estate tax is correct?

A. The interest rate on the deferred tax is determined by the prime rate in effect on the date of death.

B. The interest on the unpaid estate tax is payable over the first 10 years, after which the tax plus
interest on the balance is payable in equal installments for the last 5 years.

C. Under certain circumstances, the estate will forfeit its right to tax deferral, and all the remaining
unpaid estate tax will become due and payable immediately.

D. To qualify for the tax deferral, the closely held business must represent more than 50 percent of the
value of the decedent's adjusted gross estate.

Answer: C

QUESTION NO: 242

Which of the following areas of consideration present common ethical issues for the estate planner?

A. Contractuality

B. Conformity

C. Conservancy

D. Cooperation

Answer: D
QUESTION NO: 243

Which of the following items of property will be included in a decedent's gross estate for federal estate
tax purposes?

1. The value of property subject to a general power of appointment that the decedent possessed at
death

2. The value of all gratuitous lifetime transfers of property made within 3 years of death

A. 1 only

B. 2 only

C. Neither 1 nor 2

D. Both 1 and 2

Answer: A

QUESTION NO: 244

Mr. Barlow died early this year. Under the terms of his will he left all his real estate and tangible
personal property to his son. All the remainder of his probate estate was left to his wife, Mrs. Barlow.
The following is a list of Mr. Barlow's probate assets and their fair market values at the time of his death:

 Commercial real estate $150,000


 Furniture and fixtures 75,000
 Listed common stock 300,000
 Other securities 200,000
In addition, Mr. Barlow also owned a $100,000 life insurance policy on his life with Mrs. Barlow
designated as beneficiary. Based on this information, what is the amount of property in Mr. Barlow's
estate qualifying for the federal estate tax marital deduction?

A. $400,000

B. $825,000

C. $600,000

D. $500,000

Answer: C

QUESTION NO: 245

Which of the following statements concerning a simple trust is (are) correct?

1. All income must be distributed as earned to the person or persons who are beneficiaries of the trust.

2. Principal and income may be distributed to a qualified charity.

A. Both 1 and 2

B. Neither 1 nor 2

C. 1 only

D. 2 only

Answer: C
QUESTION NO: 246

Mr. Allen died early this year survived by his spouse Mrs. Allen. Among the items of family property are:

1.A $200,000 life insurance policy on Mr. Allen's life with Mrs. Allen designated as beneficiary. Mrs. Allen
has been the owner of the policy ever since it was issued 4 years ago.

2.The family residence with a fair market value of $300,000. Mr. and Mrs. Allen own the residence
jointly with the right of survivorship even though Mr. Allen purchased it with his separate funds.

3.A $20,000 bank account. Mr. and Mrs. Allen own the account jointly with the right of survivorship
even though Mrs. Allen made all the deposits.

What amount of the family property will be included in Mr. Allen's gross estate for federal estate tax
purposes?

A. $350,000

B. $300,000

C. $360,000

D. $160,000

Answer: D

QUESTION NO: 247

All the following statements concerning a power of appointment are correct EXCEPT:
A. The appointee of a power of appointment is the party who exercises the power.

B. The possible recipients of property after the exercise of a power of appointment depend on the terms
of the power.

C. Failure to exercise a power of appointment is known as a lapse of the power.

D. The donee of a power of appointment might be restricted with respect to the time the power can be
exercised.

Answer: A

QUESTION NO: 248

Harry, Barry, and Carrie incorporated their business, HBC and received 150 shares of stock each. They
entered into a cross-purchase buy-sell agreement at this time. The agreement binds their estates to sell
their shares of stock to the surviving shareholders. Each shareholder also agrees to purchase one-half of
the shares held by the estate of the deceased shareholder. Assume Barry dies sometime later. Which of
the following statements concerning this arrangement is (are) correct?

1. When the agreement is carried out, HBC will have 300 shares of stock outstanding.

2. Barry's estate will have additional liquidity to meet expenses and distribution requirements.

A. 2 only

B. Both 1 and 2

C. 1 only

D. Neither 1 nor 2

Answer: A
QUESTION NO: 249

Which of the following transfers will be successful in removing property from a grantor's gross estate?

1. A grantor's transfer of property to a revocable trust if the grantor lives three years after the transfer.

2. A grantor's transfer of a personal residence to a qualified personal residence trust if the grantor
survives the retained interest term.

A. 1 only

B. Both 1 and 2

C. Neither 1 nor 2

D. 2 only

Answer: D

QUESTION NO: 250

Which of the following statements concerning the so-called "kiddie-tax" on unearned income of children
under age 14 is (are) correct?

1. The rules apply to earned income of the children.

2. The rules apply to trust income received by a child under age 14 only if the trust was established by
the child's parents.

A. Both 1 and 2
B. 2 only

C. Neither 1 nor 2

D. 1 only

Answer: C

QUESTION NO: 251

All the following statements concerning guardians for minors are correct EXCEPT:

A. A guardian has equitable title to the property he administers for the minor.

B. A guardian named in a deceased parent's will is not necessarily binding on the court.

C. A guardian of the person of a minor may not necessarily be the guardian of the minor's property.

D. A special guardian can be appointed by the court to protect a minor's rights in a legal proceeding.

Answer: A

QUESTION NO: 252

A man is planning to establish and fund a 20-year irrevocable trust for the benefit of his two sons, aged
19 and 22, and plans to give the trustee power to sprinkle trust income. From the standpoint of
providing federal income, gift, and estate tax savings, which of the following would be the best choice of
trustee?

A. A bank or trust company

B. The grantor's 70-year-old father

C. The grantor of the trust


D. The grantor's 22-year-old son

Answer: A

QUESTION NO: 253

A wife with substantial property in her own name would like to provide for her husband in the event of
her prior death but does not want him to be able to leave her property to a second wife if he remarries.
She would also like to take advantage of the marital deduction to save federal estate taxes. Her attorney
has advised her that qualified terminable interest property (QTIP) will accomplish her objectives. To
qualify such property for the marital deduction, all the following requirements must be met EXCEPT:

A. An irrevocable election must be made by the decedent's executor to have the property includible in
the surviving spouse's gross estate.

B. The surviving spouse must be given a general power to appoint each year at least $5,000 or 5 percent
of the value of the property.

C. The property must have been included in the decedent's gross estate for federal estate tax purposes.

D. The surviving spouse must be given a lifetime right to receive all the property's income at least
annually.

Answer: B

QUESTION NO: 254

All the following are grounds for contesting a will EXCEPT:

A. The widow was bequeathed less than her intestate share.

B. The instrument is a forgery.


C. The testator did not have testamentary capacity.

D. The testator executed a later valid will.

Answer: A

QUESTION NO: 255

To qualify the seller of property for installment sale tax treatment, the transaction must meet which of
the following conditions?

A. There must be no more than ten installments.

B. All installments must be in equal amounts of principal.

C. At least 30 percent of the purchase price must be paid in the year of sale.

D. The entire purchase price must not be paid in the taxable year of sale.

Answer: D

QUESTION NO: 256

All the following are characteristics of a buy-sell agreement EXCEPT

A. It provides for the continuation of the business

B. It provides liquidity for estate settlement needs

C. It provides a market for the business

D. It provides for easier probating of the business


Answer: D

QUESTION NO: 257

Which of the following statements concerning filing the federal estate tax return is (are) correct?

1. The estate tax return must be filed within 9 months of death unless an extension is granted by the IRS.

2. For persons dying this year, an estate tax return must be filed for gross estates plus adjusted taxable
gifts that exceed $1.5 million.

A. Neither 1 nor 2

B. 2 only

C. 1 only

D. Both 1 and 2

Answer: D

QUESTION NO: 258

Which of the following statements concerning the methods of valuing a closely held business for federal
estate tax purposes is (are) correct?

1. The capitalization-of-adjusted-earnings method uses a capitalization rate that varies inversely with
the degree of risk and rate of return.
2. The adjusted-book value method involves adjusting the asset components of a business to an
approximate fair market value for each component.

A. 2 only

B. 1 only

C. Both 1 and 2

D. Neither 1 nor 2

Answer: C

QUESTION NO: 259

The failure of an individual to have a will can result in all the following EXCEPT:

A. A surviving spouse receives only his or her elective share.

B. The decedent's state of domicile might receive the property left by the decedent.

C. Unnecessary death taxes may be imposed.

D. Testamentary gifts to charity cannot be made.

Answer: A

QUESTION NO: 260

All the following items will be included in a decedent's gross estate at their date of death value for
federal estate tax purposes EXCEPT
A. a no-refund life annuity payable to the decedent that was purchased by the decedent from a life
insurance company

B. a gratuitous lifetime transfer of property in which the decedent retained the power to revoke the
transfer with the consent of another person

C. a gratuitous transfer of property taking effect at the decedent's death

D. a gratuitous lifetime transfer in which the decedent retained a reversionary interest on the date of
death equivalent to more than 5 percent of the value of the property

Answer: A

QUESTION NO: 261

Which of the following statements concerning marital transfers to a non-U.S. citizen spouse is (are)
correct?

1. A marital deduction is automatically available as long as property is transferred outright to the non-
citizen spouse.

2. A marital deduction is automatically available if the transferor-decedent spouse is a U. S. citizen.

A. 1 only

B. Neither 1 nor 2

C. 2 only

D. Both 1 and 2

Answer: B
QUESTION NO: 262

Which of the following is an example of a taxable gift for federal gift tax purposes?

A. Instead of parents paying an outside executive $60,000, a son runs their business for 8 months
without charging a fee.

B. The parents of a married son permit their son and his family to use a summer cottage that rents for
$3,000 per month on a rent-free basis.

C. A father cancels a $50,000 note his daughter gave him when he made a loan to her 2 years ago.

D. A father gives his 19-year-old daughter a note promising to give her his Rolls Royce when she reaches
the age of 21.

Answer: C

QUESTION NO: 263

Which of the following statements concerning the imposition of state death taxes on property owned by
a decedent is (are) correct?

1. Real estate must be taxed in the state where the decedent was domiciled.

2. Intangible personal property is generally taxed in the state where the decedent was domiciled.

A. 2 only

B. Neither 1 nor 2

C. Both 1 and 2

D. 1 only
Answer: A

QUESTION NO: 264

Alan, a widower, is a retired executive with substantial assets. He wishes to provide for the financial
security of his two grandchildren since their father, Alan's son, has always managed money poorly. This
year Alan would like each grandchild to receive a substantial gift. Which of the following statements
concerning the generation-skipping transfer tax (GSTT) on these gifts is (are) correct?

1. Federal estate or gift tax will not be imposed if the gift is otherwise subject to the GSTT.

2. Assuming no prior gifts, Alan can gift a cumulative total of (not including the annual exclusion) $1.5
million to his grandchildren without the imposition of the GSTT.

A. Both 1 and 2

B. 1 only

C. Neither 1 nor 2

D. 2 only

Answer: D

QUESTION NO: 265

A man is planning to establish and fund an irrevocable trust for the benefit of his two sons, ages 19 and
22, and plans to give the trustee power to sprinkle trust income. From the standpoint of providing
federal income, gift, and estate tax savings, which of the following would be a suitable trustee?
1. The grantor of the trust

2. The grantor's 22-year-old son

A. 2 only

B. Neither 1 nor 2

C. 1 only

D. Both 1 and 2

Answer: B

QUESTION NO: 266

All the following transfers are subject to the generation-skipping transfer tax (GSTT) EXCEPT:

A. A direct cash gift of $50,000 from a grandparent to his grandchild if such grandchild's parents are still
alive.

B. A distribution to a grandchild from a sprinkle trust created by a grandparent to benefit both skip and
non-skip beneficiaries.

C. A termination of a trust at the death of the nonskip life income beneficiary with the remainder
distributed solely to skip persons.

D. A direct cash payment of $28,000 from a grandparent to a private prep school to cover the tuition
costs for her grandchild.

Answer: D
QUESTION NO: 267

A widow made the following cash gifts during the current year:

 Donee Amount of Gift


 A qualified charity $40,000
 A close friend 30,000
 Her sister 5,000
 Her daughter 15,000
 Her brother 10,000

The total amount of the taxable gifts made this year was

A. $23,000

B. $45,000

C. $52,000

D. $95,000

Answer: A

QUESTION NO: 268

Many trust instruments provide for the removal of the original trustee. Valid reasons for removing the
original trustee include which of the following?

1. A shift in trust situs is desirable because of changes in law.

2. The beneficiary has moved his or her residence to a distant state.


A. 1 only

B. Both 1 and 2

C. Neither 1 nor 2

D. 2 only

Answer: B

QUESTION NO: 269

An individual who is a resident of State W is also the sole proprietor of a business located in State W. He
owns real property located in State X that is used by the proprietorship. While on vacation in State Y, the
individual meets an untimely death. Under the terms of his will, his entire estate is bequeathed to a
resident of State Z. Which state will tax the real property used by the proprietorship?

A. State W

B. State Z

C. State X

D. State Y

Answer: C

QUESTION NO: 270

Which of the following is an example of a taxable gift for federal gift tax purposes?

A. The parents of a married son permit their son and his family to use a summer cottage that rents for
$3,000 per month on a rent-free basis.
B. A father gives his 19-year-old daughter a note promising to give her his Rolls Royce when she reaches
the age of 21.

C. Instead of parents paying an outside executive $60,000, a son runs their business for 8 months
without charging a fee.

D. A father cancels a $50,000 note his daughter gave him when he made a loan to her 2 years ago.

Answer: D

QUESTION NO: 271

A wealthy individual might consider selling a substantially appreciated property interest in an


installment sale for which of the following reasons?

1. To spread the taxable gain inherent in the property over the period of the installments

2. To provide a buyer who lacks the requisite funds for a lump-sum purchase with the ability to finance
the acquisition

A. 2 only

B. Both 1 and 2

C. Neither 1 nor 2

D. 1 only

Answer: B

QUESTION NO: 272


Which of the following statements concerning a grantor-retained annuity trust (GRAT) is (are) correct?

1. The grantor is taxed on trust income during the retained term.

2. The grantor makes an irrevocable transfer to the remainderperson(s) when the trust is created.

A. Neither 1 nor 2

B. 2 only

C. 1 only

D. Both 1 and 2

Answer: D

QUESTION NO: 273

A married man is the sole owner of a small business with an estate tax value of $500,000. In addition, he
and his wife own an office building as joint tenants with right of survivorship which they purchased five
years ago. The building has an estate tax value of $1,500,000. They are considering dissolving the joint
tenancy and retitling the building in the name of the husband as sole owner. Which of the following
statements concerning this action is (are) correct?

1. If the husband dies first, it would be easier to qualify his estate for a Section 303 redemption of his
business interest.

2. If the husband dies first, the probate costs of his estate could be increased.

A. Neither 1 nor 2
B. 1 only

C. Both 1 and 2

D. 2 only

Answer: D

QUESTION NO: 274

All of the following statements concerning the use of an irrevocable life insurance trust to solve liquidity
problems of the insured's estate are correct EXCEPT:

A. The grantor should avoid obtaining incidents of ownership in the policy.

B. The grantor is the most appropriate choice for trustee.

C. Gift taxes can be avoided for premium contributions made by the grantor.

D. The grantor can avoid inclusion of the corpus in his or her gross estate.

Answer: B

QUESTION NO: 275

Which of the following acts by a person other than a lawyer is (are) clearly an unauthorized practice of
law?

1. A trust officer gives a client advice about the taxation of a trust.

2. A CPA designs an estate plan for presentation to a client.


A. Neither 1 nor 2

B. 1 only

C. 2 only

D. Both 1 and 2

Answer: A

QUESTION NO: 276

The federal gift tax is

A. a tax on the right of the donee to receive the property

B. levied directly on the gift

C. a flat tax based on gifts made in any taxable year

D. a tax on the right of the donor to make the gift

Answer: D

QUESTION NO: 277

Alan, a widower, is a retired executive with substantial assets. He wishes to provide for the financial
security of his two grandchildren since their father, Alan's son, has always managed money poorly. This
year Alan would like to provide each grandchild with a substantial gift. Which of the following
statements concerning the impact of the generation-skipping transfer tax (GSTT) on these transfers is
correct?
A. A direct gift of $1.5 million to each grandchild made during Alan's lifetime is exempt from GSTT.

B. Any GSTT applicable to the gifts is imposed at a marginal rate based on Alan's wealth.

C. Federal estate or gift tax will also be imposed on the gifts that are subject to the GSTT.

D. Alan could avoid all GSTT by holding all his property until death and providing direct bequests for the
grandchildren in his will.

Answer: C

QUESTION NO: 278

Which of the following acts by a person other than a lawyer is (are) clearly an unauthorized practice of
law?

1. A trust officer gives a client advice about the taxation of a trust.

2. A CPA designs an estate plan for presentation to a client.

A. 1 only

B. 2 only

C. Neither 1 nor 2

D. Both 1 and 2

Answer: C

QUESTION NO: 279


All the following factors are important in assessing liquidity needs in estate planning EXCEPT the

A. ages of the residuary estate beneficiaries

B. types of assets that comprise the estate

C. marital status of the testator

D. projected estate tax liability

Answer: A

QUESTION NO: 280

A widower dies leaving a net probate estate of $300,000. At the time of his death, his descendants are
as follows:

A son, Joe, who has no children;

A deceased daughter, Mary, whose two children, Irene and Sally, survive; and

A daughter, Anne, who has one child, Harry

Assuming that the widower's will provides for the distribution of his assets in equal shares to his
children, per stirpes, which of the following correctly states the amounts each descendant will receive?

A. $60,000 to Joe, $60,000 to Irene, $60,000 to Sally, $60,000 to Anne, and $60,000 to Harry

B. $100,000 to Joe, $50,000 to Irene, $50,000 to Sally, $50,000 to Anne, and $50,000 to Harry

C. $75,000 to Joe, $75,000 to Irene, $75,000 to Sally, and $75,000 to Anne


D. $100,000 to Joe, $50,000 to Irene, $50,000 to Sally, and $100,000 to Anne

Answer: D

QUESTION NO: 281

Losses resulting from which of the following occurrences constitutes a permissible deduction from a
decedent's gross estate to determine the adjusted gross estate?

1. Unreimbursed losses of estate assets due to theft.

2. Unreimbursed losses of estate assets due to a storm.

A. Both 1 and 2

B. Neither 1 nor 2

C. 2 only

D. 1 only

Answer: A

QUESTION NO: 282

All the following items are deductions from a decedent's gross estate in determining his adjusted gross
estate EXCEPT

A. state death taxes


B. estate administration expenses

C. claims against the estate

D. attorney fees

Answer: A

QUESTION NO: 283

Which of the following statements concerning the imposition of state death taxes on property owned by
a decedent is (are) correct?

1. Real estate must be taxed in the state where the decedent was domiciled.

2. Intangible personal property is generally taxed in the state where the decedent was domiciled.

A. 2 only

B. 1 only

C. Neither 1 nor 2

D. Both 1 and 2

Answer: A

QUESTION NO: 284

A man died in February of this year. Last year, when he learned that he had terminal illness, he
immediately made the following gifts and filed the required gift tax return:
Fair Market Value

Gift of listed stock to a

 qualified charity $200,000


 Gift of listed bonds to his wife 300,000
 Gift of a boat to his son 10,000
 Gift of a sports car to his daughter 10,000

What amount must be brought back to the man's estate as an adjusted taxable gift in the calculation of
his federal estate taxes?

A. $200,000

B. 0

C. $290,000

D. $520,000

Answer: B

QUESTION NO: 285

A married man died this year leaving a gross estate of $2,700,000. Some additional facts concerning his
estate are:

 Administration expenses and debts $300,000


 Marital deduction 800,000
 Applicable credit amount (2005) 555,800
 Applicable exclusion amount (2005) 1,500,000
 State death taxes payable 17,700
Under the Unified Rate Schedule for computing estate taxes if the amount with respect to which the
tentative tax to be computed is over $1,000,000 but not over $1,250,000, the tentative tax is $345,800,
plus 41 percent of the excess of such amount over $1,000,000. If the amount is over $1,250,000 but not
over $1,500,000, the tentative tax is then $448,300, plus 43 percent of the excess of such amount over
$1,250,000. If the amount is over $1,500,000 but not over $2,000,000 the tentative tax is then $555,800
plus 45% of the excess of such amount over $1,500,000. Based on these facts, the net federal tax
payable is

A. $42,865

B. 0

C. $47,065

D. $37,035

Answer: D

QUESTION NO: 286

Which of the following statements concerning ownership of property in the form of a joint tenancy with
right of survivorship is (are) correct?

1. Either real or personal property may be owned as a joint tenancy with right of survivorship.

2. Nonqualified joint tenants with right of survivorship may have unequal separate shares of the
property.

A. Neither 1 nor 2

B. Both 1 and 2

C. 1 only

D. 2 only
Answer: C

QUESTION NO: 287

All of the following statements concerning the use of an irrevocable life insurance trust to solve liquidity
problems of the insured's estate are correct EXCEPT:

A. The grantor is the most appropriate choice for trustee.

B. The grantor can avoid inclusion of the corpus in his or her gross estate.

C. The grantor should avoid obtaining incidents of ownership in the policy.

D. Gift taxes can be avoided for premium contributions made by the grantor.

Answer: A

QUESTION NO: 288

Which of the following statements concerning ownership of property held in trust is (are) correct?

1. The legal owner of property held in trust is a trustee.

2. The equitable owner of property held in trust is a beneficiary.

A. 1 only

B. Both 1 and 2

C. Neither 1 nor 2
D. 2 only

Answer: B

QUESTION NO: 289

All the following powers held by the grantor of an irrevocable trust will cause the trust assets to be
brought back into the estate of the grantor EXCEPT the power to

A. terminate the trust

B. add principal to the trust

C. designate who shall enjoy the trust income

D. change the trust remainderpersons

Answer: B

QUESTION NO: 290

Examples of terminable interests that are nondeductible for the federal estate tax marital deduction
include which of the following?

1. A property interest that would pass to someone else if the surviving spouse remarries

2. A life estate in property that would pass to another person at the death of the surviving spouse

A. 2 only
B. Both 1 and 2

C. Neither 1 nor 2

D. 1 only

Answer: B

QUESTION NO: 291

A woman is the income beneficiary of an irrevocable trust. Which of the following powers given to her
will cause all the assets in the trust to be includible in her gross estate for federal estate tax purposes?

A. The testamentary special or limited power to direct the trustee to distribute trust assets to her
children

B. The testamentary power to direct the trustee to use trust assets to pay her estate taxes

C. The power each year to direct the trustee to pay her an amount of trust assets not exceeding the
greater of $5,000 or 5 percent of the assets held by the trust

D. The power to direct the trustee to pay trust assets to her limited in amount to an ascertainable
standard relating to her health and education

Answer: B

QUESTION NO: 292

Which of the following statements concerning a power of appointment is correct?

A. The time that a special power of appointment may be exercised may not be restricted by the donor
when creating the power.
B. A limited power of appointment permits the donee of the power to exercise the power in favor of
anyone except the donee's family members.

C. A donee who possesses a special power of appointment at the time of his or her death has the
property included in the gross estate.

D. The recipients of the property after the donee exercises the power of appointment are known as the
appointees.

Answer: D

QUESTION NO: 293

The estate planning process includes all the following activities EXCEPT

A. the selection of the estate plan by the financial planner

B. evaluating a client's present estate plan

C. gathering a client's family and financial data

D. designing a client's new estate plan

Answer: A

QUESTION NO: 294

Which of the following statements concerning an installment sale is (are) correct?

1. Some portion of the purchase price must be paid in any one taxable year other than the year of sale.

2. Installments due after the seller's death are excludible from the seller's gross estate.
A. 1 only

B. Neither 1 nor 2

C. 2 only

D. Both 1 and 2

Answer: A

QUESTION NO: 295

All the following statements concerning the gift and estate tax chartiable deduction are correct EXCEPT:

A. An estate tax charitable deduction is allowed for the full value of property transferred to a qualified
charity but only if the property is included in the donor gross estate.

B. A donor is denied a charitable deduction for property that passes to a qualified charity as the result of
a qualified disclaimer if the donor original transfer was to a noncharitable donee.

C. It is possible for a charitable contribution made during the donor lifetime to generate both income
and transfer tax deductions for the donor.

D. If the donor retains an interest in property contributed to a qualified charity during lifetime, the value
of the property may be included in the donor gross estate.

Answer: B

QUESTION NO: 296

A single man with substantial assets and income is supporting his 80-year-old partially senile mother
with monthly cash gifts. He is trying to find a practical way to support his mother while at the same time
saving federal gift and income taxes without giving up ultimate control of any assets. Which of the
following courses of action will best accomplish these objectives?

A. Make her a gift of enough corporate bonds from his portfolio so that she will be able to support
herself from the interest payments

B. Make her annual gifts of enough interest income from the tax free municipal bonds in his portfolio so
that she will be able to support herself

C. Purchase corporate bonds that pay interest in an amount sufficient for her to support herself and
assign the interest payments to her

D. Make her an interest free loan with a principal amount large enough to produce sufficient income for
her support when invested in corporate bonds

Answer: B

QUESTION NO: 297

Which of the following statements concerning the joint tenancy with right of survivorship form of real
property ownership is (are) correct?

1. The property is part of the probate estate of the first tenant to die.

2. An owner can sell his interest in the property at any time without destroying the form of ownership.

A. Both 1 and 2

B. Neither 1 nor 2

C. 1 only

D. 2 only
Answer: B

QUESTION NO: 298

A married man died this year leaving a gross estate of $2,750,000. Some additional facts concerning his
estate are:

 Administration expenses and debts $ 150,000


 Marital deduction 750,000
 Applicable credit amount (2005) 555,800
 Applicable exclusion amount (2005) 1,500,000
 State death taxes payable 22,200

Under the Unified Rate Schedule for computing estate taxes if the amount with respect to which the
tentative tax to be computed is over $1,000,000 but not over $1,250,000, the tentative tax is $345,800,
plus 41 percent of the excess of such amount over $1,000,000. If the amount is over $1,250,000 but not
over $1,500,000, the tentative tax is then $448,300, plus 43 percent of the excess of such amount over
$1,250,000. If the amount is over $1,500,000 but not over $2,000,000 the tentative tax is then $555,800
plus 45% of the excess of such amount over $1,500,000. Based on these facts, the net federal estate tax
payable is

A. $163,710

B. 0

C. $151.490

D. $147,510

Answer: D

QUESTION NO: 299


All the following are conditions that must be met if an otherwise nonqualified terminable interest is to
qualify (as QTIP) for the federal estate tax marital deduction EXCEPT:

A. The surviving spouse must make a qualified disclaimer to all other property in the deceased spouse's
estate within 9 months of death.

B. The surviving spouse must be given a lifetime right to receive all the property's income at least
annually.

C. The deceased spouse's executor must make an irrevocable election to have the property includible in
the surviving spouse's gross estate.

D. No person can be given the right to direct that the property go to anyone other than the surviving
spouse as long as the surviving spouse is alive.

Answer: A

QUESTION NO: 300

A man recently died with only probate assets. Under the terms of his will, he left his entire probate
estate outright to his wife. The following are relevant facts concerning the estate:

 Gross estate $2,400,000


 Estate administration expenses 85,000
 Debts of decedent 100,000
 Allowable funeral expenses 10,000

The amount of the allowable marital deduction is

A. $2,315,000

B. $2,205,000

C. $2,305,000
D. $2,215,000

Answer: B

QUESTION NO: 301

A person dying without a will loses all the following rights EXCEPT the right to

A. give property to a charity

B. take maximum advantage of the marital deduction

C. name the person to settle the estate

D. have assets pass to heirs

Answer: D

QUESTION NO: 302

Which of the following statements concerning the imposition of state death taxes on property owned by
a decedent is (are) correct?

1. Real estate must be taxed in the state where the decedent was domiciled.

2. Intangible personal property is generally taxed in the state where the decedent was domiciled.

A. 2 only

B. Both 1 and 2
C. 1 only

D. Neither 1 nor 2

Answer: A

QUESTION NO: 303

All the following statements concerning a federal estate tax deduction for a bequest or gift to a qualified
charity are correct EXCEPT:

A. The amount of a charitable deduction may not exceed 50 percent of a decedent's adjusted gross
estate.

B. An estate may deduct the value of the remainder interest in a charitable remainder trust.

C. A life insurance policy that was assigned to a charity as a gift less than 3 years prior to the insured's
death qualifies for a charitable

deduction.

D. The amount of a charitable deduction is reduced by any taxes and administrative expenses
chargeable against the bequest.

Answer: A

QUESTION NO: 304

All the following statements concerning a federal estate tax deduction for a bequest or gift to a qualified
charity are correct EXCEPT:

A. A life insurance policy that was assigned to a charity as a gift less than 3 years prior to the insured's
death qualifies for a charitable deduction.
B. The amount of a charitable deduction is reduced by any taxes and administrative expenses chargeable
against the bequest.

C. The amount of a charitable deduction may not exceed 50 percent of a decedent's adjusted gross
estate.

D. An estate may deduct the value of the remainder interest in a charitable remainder trust.

Answer: C

QUESTION NO: 305

An individual who is a resident of State W is also the sole proprietor of a business located in State W. He
owns real property located in State X that is used by the proprietorship. While on vacation in State Y, the
individual meets an untimely death. Under the terms of his will, his entire estate is bequeathed to a
resident of State Z. Which state will tax the real property used by the proprietorship?

A. State X

B. State W

C. State Y

D. State Z

Answer: A

QUESTION NO: 306

Which of the following statements concerning property is (are) correct?

1. A mortgage on real estate is real property.


2. A tree growing on land is real property.

A. 2 only

B. 1 only

C. Neither 1 nor 2

D. Both 1 and 2

Answer: A

QUESTION NO: 307

All the following statements concerning the ownership of real property as joint tenants with right of
survivorship are correct EXCEPT:

A. If three sisters inherited property as joint tenants with right of survivorship, the entire value of the
property will be in the estate of the first sister to die.

B. If the joint tenants are husband and wife, because this is a qualified joint interest, one half the value
of the property will be in the estate of the first spouse to die regardless of which spouse contributed to
the purchase price.

C. If the joint tenants are two brothers and each contributed one half the property's purchase price, only
one half the property's value will be in the estate of the first brother to die if his executor proves that
the other brother contributed half of the purchase price.

D. If the joint tenants are brother and sister, no portion of the value of the property will be in the sister's
estate if she dies first provided her executor proves that the brother contributed all the funds.

Answer: A
QUESTION NO: 308

Believing that his death was imminent, a widower gave his son some real estate two years ago, and filed
a timely gift tax return. The widower died on January 1st of this year. Additional facts are:

Widower's basis in the real estate $200,000

Value of real estate when gifted 510,000

Value of real estate on date of death 1,000,000

Amount of gift tax paid by widower 159,500

Assuming the widower made no additional gifts to his son, all the following statements concerning this
situation are correct EXCEPT:

A. The widower recognized no capital gain for income tax purposes at the time the gift was made.

B. The gift of the real estate is included in the calculation of the widower's federal estate tax as an
adjusted taxable gift.

C. The son's income tax basis in the real estate is $1,000,000.

D. The gift tax paid is brought back into the widower's gross estate at $159,500.

Answer: C

QUESTION NO: 309

A father is considering giving his daughter a gift. For tax planning purposes, the father should give his
daughter which of the following?

A. A bond that cost him $25,000 and is now worth $10,000


B. Securities that cost him $10,000, its present fair market value, but which has a substantial potential
for appreciation

C. Raw land that cost him $1,000 and which now has a fair market value of $14,000

D. Real estate that cost him $30,000 and is now worth $110,000, subject to a $100,000 mortgage

Answer: B

QUESTION NO: 310

A father wants to accumulate funds for his 12-year-old son's college education. On the advice of his
attorney, the father establishes an IRC Section 2503(c) trust and funds it with annual gifts. All the
following statements concerning this arrangement are correct EXCEPT:

A. Any accumulated income and all trust principal must be available for distribution to the son when he
attains age 21.

B. The father's annual gift tax exclusion must be reduced by any amount used to pay college tuition
costs.

C. The trust must be irrevocable.

D. In the event of the son's death prior to age 21, trust assets must either be payable to the son's estate
or be subject to a general power of appointment held by the son.

Answer: B

QUESTION NO: 311

Believing that his death was imminent, a widower gave his son some real estate two years ago, and filed
a timely gift tax return. The widower died on January 1st of this year. Additional facts are:
 Widower's basis in the real estate $150,000
 Value of real estate when gifted 400,000
 Value of real estate on date of death 800,000
 Amount of gift tax paid by widower 121,800

Assuming the widower made no additional gifts to his son, all the following statements concerning this
situation are correct EXCEPT:

A. The gift tax paid is brought back into the widower's gross estate at $121,800.

B. The gift of the real estate is included in the calculation of the widower's federal estate tax as an
adjusted taxable gift.

C. The son's income tax basis in the real estate is $800,000.

D. The widower recognized no gain for income tax purposes at the time the gift was made.

Answer: C

QUESTION NO: 312

A woman is the income beneficiary of an irrevocable trust. All the following powers held by her will
cause all the assets in the trust to be includible in her gross estate for federal estate tax purposes
EXCEPT

A. the power to direct the trustee to distribute trust corpus to her

B. the testamentary special or limited power to direct the trustee to distribute trust assets to her
children

C. the testamentary power to direct the trustee to pay trust assets to her estate

D. the testamentary power to direct the trustee to use trust assets to pay her estate taxes

Answer: B
QUESTION NO: 313

All the following are conditions that must be met if an otherwise nonqualified terminable interest is to
qualify (as QTIP) for the federal estate tax marital deduction EXCEPT:

A. No person can be given the right to direct that the property go to anyone other than the surviving
spouse as long as the surviving spouse is alive.

B. The deceased spouse's executor must make an irrevocable election to have the property includible in
the surviving spouse's gross estate.

C. The surviving spouse must be given a lifetime right to receive all the property's income at least
annually.

D. The surviving spouse must make a qualified disclaimer to all other property in the deceased spouse's
estate within 9 months of death.

Answer: D

QUESTION NO: 314

The decedent, Z, died this year. The facts concerning Z estate are:

 Gross estate $3,200,000


 Marital deduction $1,100,000
 Charitable deduction 80,000
 Gifts made after 1976 115,000
 State death taxes payable 215,000

What is Z taxable estate?


A. $1,805,000

B. $1,920,000

C. $1,590,000

D. $1,690,000

Answer: A

QUESTION NO: 315

All the following statements concerning a typical pour-over trust are correct EXCEPT:

A. It is a device to consolidate all a decedent's assets to simplify administration.

B. Properly drawn, it eliminates the need to file a federal estate tax return.

C. The trust is created during the lifetime of the grantor.

D. The trust is revocable during the lifetime of the grantor.

Answer: B

QUESTION NO: 316

When the owner of a closely held business dies, the payment of a portion of the federal estate tax may
be deferred for a period of several years if the estate otherwise qualifies under the provisions of IRC
Section 6166. Which of the following statements concerning this deferral of federal estate tax is correct?

A. The interest rate on the deferred tax is determined by the prime rate in effect on the date of death.

B. The interest on the unpaid estate tax is payable over the first 10 years, after which the tax plus
interest on the balance is payable in equal installments for the last 5 years.
C. To qualify for the tax deferral, the closely held business must represent more than 50 percent of the
value of the decedent's adjusted gross estate.

D. Under certain circumstances, the estate will forfeit its right to tax deferral, and all the remaining
unpaid estate tax will become due and payable immediately.

Answer: D

QUESTION NO: 317

A father is considering giving his daughter a gift. For tax planning purposes, the father should give his
daughter which of the following?

A. Raw land that cost him $10,000, its present fair market value, but which has a substantial potential
for appreciation

B. A bond that cost him $15,000 and is now worth $10,000

C. Real estate that cost him $40,000 and is now worth $120,000, subject to a $110,000 mortgage

D. Stock that cost him $10,000 and which now has a fair market value of $20,000

Answer: A

QUESTION NO: 318

Which of the following types of real property ownership will be deemed to be a tenancy in common?

A. Two brothers are equal partners in a general partnership that owns a piece of real property used in
the partnership business.

B. Two brothers own equal fractional interests in a piece of real property and at the death of one of the
brothers the survivor will own the entire piece of property.
C. Two brothers own equal undivided interests in a piece of real property, with each brother being able
to divest himself of his interest by sale, gift, or will.

D. Two brothers own equal amounts of all the common stock in a corporation, the only asset of which is
real property.

Answer: C

QUESTION NO: 319

Which of the following statements concerning the generation-skipping transfer tax (GSTT) is correct?

A. The GSTT rate applicable to a transfer depends upon the amount of the gift.

B. The lifetime exemption shelters a maximum of $1 million of transfers to grandchildren from GSTT for
the current year.

C. Tuition payments made by a grandparent directly to a university for a grandchild's education are
exempt from GSTT.

D. The annual exclusion against GSTT shelters gifts by a grandparent to a trust benefitting multiple
grandchildren.

Answer: C

QUESTION NO: 320

A married man died this year leaving a gross estate of $3,200,000. Additional facts concerning his estate
are:

 Administration expenses and debts $ 250,000


 Marital deduction 1,200,000
 Applicable credit amount (2005) 555,800
 Applicable exclusion amount (2005) 1,500,000
 State death taxes payable 20,400

Under the Unified Rate Schedule for computing estate taxes if the amount with respect to which the
tentative tax to be computed is over $1,000,000 but not over $1,250,000, the tentative tax is $345,800,
plus 41 percent of the excess of such amount over $1,000,000. If the amount is over $1,250,000 but not
over $1,500,000, the tentative tax is then $448,300, plus 43 percent of the excess of such amount over
$1,250,000. If the amount is over $1,500,000 but not over $2,000,000, the tentative tax is then
$555,800 plus 45% of the excess of such amount over $1,500,000. Based on these facts, the net federal
estate tax payable is

A. 0

B. $123,720

C. $103,320

D. $128,280

Answer: C

QUESTION NO: 321

Which of the following provisions is (are) generally common to all buy-sell agreements?

1. Provisions for modification of the agreement.

2. Provisions for lifetime business interest transfer restrictions.

A. Both 1 and 2

B. 1 only
C. 2 only

D. Neither 1 nor 2

Answer: A

QUESTION NO: 322

Which of the following members of the estate planning team is responsible for assuring that the client's
intentions are expressed in documents that will carry out the final plan?

A. The trust officer

B. The lawyer

C. The investment counselor

D. The life underwriter

Answer: B

QUESTION NO: 323

All the following powers held by the grantor of an irrevocable trust will cause the trust assets to be
brought back into the estate of the grantor EXCEPT the power to

A. designate who shall enjoy the trust income

B. add principal to the trust

C. change the trust remainderpersons

D. terminate the trust


Answer: B

QUESTION NO: 324

All the following statements concerning revocable trusts are correct EXCEPT:

A. A transfer to a revocable trust is treated as an incomplete gift for gift tax purposes.

B. A transfer to a revocable trust changes the income tax picture of the grantor.

C. Property transferred to a revocable trust typically avoids being included in the probate estate of the
grantor.

D. Revocable trusts are created and operate before the death of the settlor.

Answer: B

QUESTION NO: 325

Which of the following statements concerning federal gift, estate, and income taxes is (are) correct?

1. A taxable gift of income-producing property to a donee automatically transfers income tax liability on
the gifted property to the donee.

2. Once part or all of a taxable gift is made to a trust, the property can no longer be includible in the
donor gross estate.

A. Neither 1 nor 2

B. 2 only
C. 1 only

D. Both 1 and 2

Answer: A

QUESTION NO: 326

Which of the following statements concerning charitable remainder unitrusts is correct?

A. The remainder interest is paid to the qualified charity after a term of years not greater than 15 years.

B. A fixed percentage of not less than 10 percent of the net fair market value of the trust assets is paid
to the noncharitable beneficiaries.

C. No further contributions may be made to a unitrust after the initial payment.

D. The net fair market value of the trust assets are revalued annually.

Answer: D

QUESTION NO: 327

Which of the following statements concerning federal gift, estate, and income taxes is (are) correct:

1. A taxable gift of income-producing property automatically transfers income tax liability to the donee.

2. The value of gifts made within 3 years of death cannot be brought back into the donor's gross estate.

A. Neither 1 nor 2
B. 1 only

C. 2 only

D. Both 1 and 2

Answer: A

QUESTION NO: 328

Mr. Allen died early this year survived by his spouse Mrs. Allen. Among the items of family property are:

1.A $300,000 life insurance policy on Mr. Allen's life with Mrs. Allen designated as beneficiary. Mrs. Allen
has been the owner of the policy ever since it was issued 4 years ago.

2.The family residence with a fair market value of $400,000. Mr. and Mrs. Allen own the residence
jointly with the right of survivorship even though Mr. Allen purchased it with his separate funds.

3.A $40,000 bank account. Mr. and Mrs. Allen own the account jointly with the right of survivorship
even though Mrs. Allen made all the deposits.

What amount of the family property will be included in Mr. Allen's gross estate for federal estate tax
purposes?

A. $500,000

B. $520,000

C. $220,000

D. $400,000
Answer: C

QUESTION NO: 329

All the following statements concerning irrevocable trusts are correct EXCEPT:

A. An irrevocable trust is treated as a completed gift for estate, gift, and income tax purposes.

B. An irrevocable trust is one which the grantor cannot terminate and reclaim the trust property.

C. Property transferred to an irrevocable trust will not be included in the grantor's probate property.

D. A transfer of property to an irrevocable trust will be ineffective for the purpose of reducing the
grantor's gross estate.

Answer: D

QUESTION NO: 330

Which of the following statements concerning charitable guaranteed annuity interests is (are) correct?

1. To qualify for an estate tax charitable deduction, guaranteed annuity interests must be made in trust.

2. These interests refer to the charity right to receive a determinable income amount at least annually
for a specific term or life (lives) or one or more noncharitable beneficiaries.

A. 2 only

B. Neither 1 nor 2

C. Both 1 and 2
D. 1 only

Answer: A

QUESTION NO: 331

A married man is the sole owner of a small business with an estate tax value of $500,000. In addition, he
and his wife own an office building as joint tenants with right of survivorship which they purchased five
years ago. The building has an estate tax value of $1,500,000. They are considering dissolving the joint
tenancy and retitling the building in the name of the husband as sole owner. Which of the following
statements concerning this action is (are) correct?

1. If the husband dies first, it would be easier to qualify his estate for a Section 303 redemption of his
business interest.

2. If the husband dies first, the probate costs of his estate could be increased.

A. 2 only

B. 1 only

C. Neither 1 nor 2

D. Both 1 and 2

Answer: A

QUESTION NO: 332

A number of states have passed statutes governing "living wills." Which of the following statements
concerning a living will is (are) correct?
1. A living will is an alternative to an inter vivos trust under certain circumstances.

2. A living will is an oral will made by the testator during a final illness when it is impossible to write one.

A. Neither 1 nor 2

B. 1 only

C. 2 only

D. Both 1 and 2

Answer: A

QUESTION NO: 333

Which of the following statements concerning state death taxes is correct?

A. A state inheritance tax is imposed on the right of the deceased to leave property to heirs.

B. A state estate tax is imposed on the right of heirs to receive property from the deceased.

C. State estate and inheritance taxes are generally imposed at the same rate regardless of the
relationship of the deceased to the beneficiary.

D. A deduction for the full amount of state death taxes paid by a decedent estate is allowed on a
decedent federal estate tax return.

Answer: D
QUESTION NO: 334

Which of the following statements concerning antenuptial agreements is (are) correct?

1. They are frequently used prior to second marriages.

2. They can affect a surviving spouse's right to receive an intestate share of a deceased spouse's estate.

A. Both 1 and 2

B. Neither 1 nor 2

C. 1 only

D. 2 only

Answer: A

QUESTION NO: 335

Which of the following statements concerning property is (are) correct?

1. Personal property is all property that is not considered to be real property.

2. A mortgage on real property is treated as real property.

A. 1 only

B. 2 only

C. Both 1 and 2
D. Neither 1 nor 2

Answer: A

QUESTION NO: 336

The Decedent, T, died this year. The facts concerning T estate are:

 Gross estate $2,700,000


 Marital deduction 900,000
 Charitable deduction 110,000
 Gifts made after 1976 130,000
 State death taxes payable 165,000

What is T taxable estate?

A. $1,395,000

B. $1,525,000

C. $1,655,000

D. $1,285,000

Answer: B

QUESTION NO: 337

Which of the following statements concerning the methods of valuing a closely held business for federal
estate tax purposes is (are) correct?
1. The capitalization-of-adjusted-earnings method uses a capitalization rate that varies inversely with
the degree of risk and rate of return.

2. The adjusted-book value method involves adjusting the asset components of a business to an
approximate fair market value for each component.

A. 1 only

B. 2 only

C. Both 1 and 2

D. Neither 1 nor 2

Answer: C

QUESTION NO: 338

The following are facts concerning a decedent's estate:

 Taxable estate $2,000,000


 Pre-1977 taxable gifts 500,000
 Post-1976 adjusted taxable gifts 300,000
 Post-1976 gifts made to a qualified charity 200,000

The tentative tax base of this estate is

A. $2,300,000

B. $ 400,000
C. $2,500,000

D. $2,000,000

Answer: A

QUESTION NO: 339

Which of the following life insurance settlement options will qualify for the federal estate tax marital
deduction?

1. Proceeds left to the surviving spouse under the interest option, with interest payable to the surviving
spouse who has the unrestricted right to withdraw proceeds and with any proceeds not withdrawn
payable equally to her children per stirpes

2. Proceeds left to the surviving spouse under an installment option, with any installments remaining at
her death to be commuted and paid to her estate

A. Both 1 and 2

B. 2 only

C. 1 only

D. Neither 1 nor 2

Answer: A

QUESTION NO: 340


Which of the following statements concerning marital transfers to a non-U.S. citizen spouse is (are)
correct?

1. A marital deduction is automatically available as long as property is transferred outright to the non-
citizen spouse.

2. A marital deduction is automatically available if the transferor-decedent spouse is a U. S. citizen.

A. 2 only

B. 1 only

C. Neither 1 nor 2

D. Both 1 and 2

Answer: C

QUESTION NO: 341

Which of the following statements concerning ownership of property under a tenancy by the entirety is
correct?

A. The property will be in the probate estate of the first joint tenant to die.

B. One tenant can freely transfer his or her property interest to a third person.

C. It is a form of property ownership that applies only to personal property.

D. It is a form of property ownership available only to married persons.

Answer: D
QUESTION NO: 342

A father and son have been farming land owned by the father for the past 12 years. Just prior to his
death, the father was offered $900,000 for his farm because of its possible use as a shopping center. The
son would like to continue to farm the land if it can be included in his father's estate at its current use
value. Additional facts are:

1.Average annual gross rentals from nearby farms of similar acreage are $36,000.

2.Average annual state and local real estate taxes on the farm are $4,000.

3.The interest rate for loans from the Federal Land Bank is 8 percent.

For federal estate tax purposes, the farm method valuation formula would result in a current use value
for the farm of

A. $400,000

B. $500,000

C. $600,000

D. $300,000

Answer: A

QUESTION NO: 343

Which of the following statements concerning certain types of property interests is (are) correct?
1. The person or entity who has title to the property is the legal owner of the property.

2. The person who has the right to all income earned on the property is the beneficial or equitable
owner of the property.

A. Neither 1 nor 2

B. 1 only

C. 2 only

D. Both 1 and 2

Answer: D

QUESTION NO: 344

All the following statements concerning the gift and estate tax chartiable deduction are correct EXCEPT:

A. An estate tax charitable deduction is allowed for the full value of property transferred to a qualified
charity but only if the property is included in the donor gross estate.

B. It is possible for a charitable contribution made during the donor lifetime to generate both income
and transfer tax deductions for the donor.

C. If the donor retains an interest in property contributed to a qualified charity during lifetime, the value
of the property may be included in the donor gross estate.

D. A donor is denied a charitable deduction for property that passes to a qualified charity as the result of
a qualified disclaimer if the donor original transfer was to a noncharitable donee.

Answer: D
QUESTION NO: 345

All the following are grounds for contesting a will EXCEPT:

A. The widow was bequeathed less than her intestate share.

B. The instrument is a forgery.

C. The testator did not have testamentary capacity.

D. The testator executed a later valid will.

Answer: A

QUESTION NO: 346

All the following statements concerning revocable trusts are correct EXCEPT:

A. A transfer to a revocable trust changes the income tax picture of the grantor.

B. A transfer to a revocable trust is treated as an incomplete gift for gift tax purposes.

C. Property transferred to a revocable trust typically avoids being included in the probate estate of the
grantor.

D. Revocable trusts are created and operate before the death of the settlor.

Answer: A

QUESTION NO: 347

To determine whether a taxable gift has been made, the IRS focuses on all the following factors EXCEPT:
A. Was the value of the gift property in excess of the annual per-donee exclusion?

B. Was the transferred property real property or personal property?

C. Did the donor absolutely, irrevocably, and currently divest himself of dominion and control over the
property?

D. Was the property transferred for less than an adequate and full consideration in money or money's
worth?

Answer: B

QUESTION NO: 348

In which of the following situations will the grantor be taxed on income from trust property.

1. The grantor of a trust gives one of the trust beneficiaries the right to add or delete beneficiaries.

2. An adverse party to the grantor holds the power to determine the timing of trust distributions to the
beneficiaries.

A. Both 1 and 2

B. 2 only

C. 1 only

D. Neither 1 nor 2

Answer: D
QUESTION NO: 349

Which of the following are ways of passing property from a deceased spouse to a surviving spouse so
that the property will qualify for the federal estate tax marital deduction?

1. When the surviving spouse receives the property by electing to take against the deceased spouse's
will

2. When the surviving spouse receives the property as a consequence of the qualified disclaimer of
another beneficiary

A. 2 only

B. 1 only

C. Neither 1 nor 2

D. Both 1 and 2

Answer: D

QUESTION NO: 350

Which of the following statements concerning the inclusion in a decedent-employee's gross estate of a
lump-sum distribution from a qualified retirement plan to a beneficiary other than the employee's
estate is (are) correct?

1. Lump-sum distributions of payments attributable to the employer's contributions are excluded from
the gross estate.

2. Lump-sum distributions of payments attributable to the decedent-employee's contributions are


excluded from the gross estate.
A. Both 1 and 2

B. Neither 1 nor 2

C. 1 only

D. 2 only

Answer: B

QUESTION NO: 351

An executor elects to value the assets of the estate at the alternative valuation date 6 months after
death. Which of the following statements concerning the estate tax value of assets included in this
estate is correct?

A. Property sold before the alternate valuation date is valued at the alternate valuation date.

B. Property that has increased in value since the date of death may be valued at the date of death if the
executor so elects.

C. Property distributed under the will before the alternate valuation date is valued at the date of death.

D. An annuity included in the gross estate that diminishes with the mere passage of time is includible at
the date of death value.

Answer: D

QUESTION NO: 352

Which of the following statements concerning ownership of property under a tenancy by the entirety is
correct?
A. It is a form of property ownership available only to married persons.

B. It is a form of property ownership that applies only to personal property.

C. One tenant can freely transfer his or her property interest to a third person.

D. The property will be in the probate estate of the first joint tenant to die.

Answer: A

QUESTION NO: 353

A father and son have been farming land owned by the father for the past 12 years. Just prior to his
death, the father was offered $900,000 for his farm because of its possible use as a shopping center. The
son would like to continue to farm the land if it can be included in his father's estate at its current use
value. Additional facts are:

1.Average annual gross rentals from nearby farms of similar acreage are $36,000.

2.Average annual state and local real estate taxes on the farm are $4,000.

3.The interest rate for loans from the Federal Land Bank is 8 percent.

For federal estate tax purposes, the farm method valuation formula would result in a current use value
for the farm of

A. $600,000

B. $300,000

C. $500,000
D. $400,000

Answer: D

QUESTION NO: 354

Tax benefits of making lifetime gifts in excess of the gift tax annual exclusion include all the following
EXCEPT:

A. Appreciation in the value of a gift of real property after the date of the gift increases the donor's
federal estate tax liability.

B. Gift taxes are payable at the same tax rate as estate taxes.

C. Income taxes can be saved if a high-income donor gives income-producing property to a low-income
donee.

D. The gift tax paid on a gift made more than 3 years prior to the death of the donor avoids inclusion the
donor's gross estate.

Answer: A

QUESTION NO: 355

The failure of an individual to have a will can result in which of the following?

1. The state will determine the disposition of the individual's probate estate.

2. The decedent's preference for a personal representative, guardian, and other fiduciary roles may be
ignored.
A. Neither 1 nor 2

B. 2 only

C. 1 only

D. Both 1 and 2

Answer: D

QUESTION NO: 356

The following are facts concerning a decedent's estate:

 Taxable estate $1,800,000


 Pre-1977 taxable gifts 100,000
 Post-1976 adjusted taxable gifts 150,000
 Post-1976 gifts made to a qualified charity 200,000

The tentative tax base of this estate is

A. $2,150,000

B. $1,650,000

C. $1,800,000

D. $1,950,000

Answer: D
QUESTION NO: 357

Which of the following actions on the part of a trustee is a breach of his duties?

A. Purchasing securities in good faith from a third party just prior to a sharp decrease in their value

B. Investing all trust assets in securities that favor the income beneficiaries to the detriment of
remainderpersons

C. Placing cash from the sale of securities in a high yield money market fund pending a decision to invest
the funds elsewhere

D. Purchasing assets for personal use from the trust at their fair market value with the approval of all
beneficiaries

Answer: B

QUESTION NO: 358

All the following statements concerning installment sale tax treatment are correct EXCEPT:

A. The seller must pay income tax on the interest portion of each installment.

B. If the seller has a gain, the basis portion of each installment is received tax free.

C. Installments due after the seller's death are excludible from the seller's gross estate.

D. The entire purchase price may be fully paid in any one taxable year other than the year in which the
property is sold.

Answer: C

QUESTION NO: 359


Tax benefits of making lifetime gifts in excess of the gift tax annual exclusion include which of the
following?

1. The gift tax paid on a gift made more than 3 years prior to the death of the donor is not brought back
into the donor's estate.

2. Such gifts make use of the lifetime applicable credit amount against gift taxes which is wasted if the
property is retained until the donor's death.

A. 1 only

B. Both 1 and 2

C. 2 only

D. Neither 1 nor 2

Answer: A

QUESTION NO: 360

On the advice of their attorney and accountant, Betsy and John have decided to make substantial
transfers. They would like to pass most of their considerable wealth to their grandchildren. Which of the
following statements concerning gifts made to their grandchildren is correct?

A. Betsy and John may elect to split any GSTT transfers to the grandchildren.

B. The value of Betsy and John's GSTT exemption amounts are slightly increased when used at death
rather than during lifetime.

C. The GSTT annual exclusion may be utilized by Betsy and John for each grandchild during lifetime and
at death.

D. The GSTT annual exclusion is unavailable for years in which Betsy and John make tuition gifts for the
grandchildren.
Answer: A

QUESTION NO: 361

All the following items of property will be included in a decedent's gross estate for federal estate tax
purposes EXCEPT

A. the value of property subject to a general power of appointment that the decedent possessed at
death

B. the value of a gratuitous lifetime transfer in which the decedent retained a reversionary interest on
the date of death worth more than 5 percent of the value of the property and which the donee must
survive the decedent to possess

C. the value of a gratuitous lifetime transfer in which the decedent retained the right for life to receive
the income from the property

D. the value of all gratuitous lifetime transfers of property made within 3 years of death

Answer: D

QUESTION NO: 362

The personal representative of a decedent has the duty to file all the following tax returns EXCEPT

A. the federal estate tax return

B. the decedent's final income tax return

C. the estate's income tax return

D. the surviving spouse's income tax return for the year of death
Answer: D

QUESTION NO: 363

Which of the following statements concerning the gift or estate tax charitable deduction is (are) correct?

1. A donor is denied a charitable deduction for property that passes to a qualified charity as the result of
a qualified disclaimer if the donor original transfer was to a noncharitable donee.

2. A decedent-spouse estate may obtain both marital and charitable deductions for interests
contributed to a charitable remainder trust when the surviving spouse is the only noncharitable income
beneficiary for life.

A. Both 1 and 2

B. 1 only

C. 2 only

D. Neither 1 nor 2

Answer: C

QUESTION NO: 364

All the following powers held by the grantor of an irrevocable trust will cause the trust assets to be
brought back into the estate of the grantor EXCEPT the power to

A. designate who shall enjoy the trust income


B. add principal to the trust

C. change the trust remainderpersons

D. terminate the trust

Answer: B

QUESTION NO: 365

For estate tax purposes which of the following is (are) a form of charitable gifts that may qualify for the
charitable deduction?

1. Partial interests

2. Guaranteed annuity interests

A. 2 only

B. Neither 1 nor 2

C. 1 only

D. Both 1 and 2

Answer: D

QUESTION NO: 366

A taxable gift has been made in which of the following situations?


1. A father manages his disabled son's business for a year without compensation since a replacement
manager would have cost $25,000.

2. A father verbally promises his 21-year-old daughter that he will give her his antique Mercedes when
she graduates from college next year.

A. Neither 1 nor 2

B. Both 1 and 2

C. 1 only

D. 2 only

Answer: A

QUESTION NO: 367

Under the terms of his will, a man left his residuary estate to a testamentary trust for the benefit of his
wife. Which of the following powers with respect to the trust will cause all the trust principal to be
includible in the gross estate of the widow for federal estate tax purposes?

A. The power of the widow to direct the trustee to use trust assets to pay her personal debts

B. The power of the trustee in its sole discretion to distribute trust assets to the widow for any reason
satisfactory to the trustee

C. The testamentary limited or special power of the widow to direct the trustee to distribute trust assets
to her heirs.

D. The power of the widow each year to direct the trustee to pay her the greater of 5 percent of the
trust principal or $5,000

Answer: A
QUESTION NO: 368

Which of the following statements concerning estates and trusts is (are) correct?

1. Both estates and trusts come into being by operation of law.

2. The personal representative of an estate and the trustee have similar fiduciary responsibilities.

A. Both 1 and 2

B. 2 only

C. Neither 1 nor 2

D. 1 only

Answer: B

QUESTION NO: 369

Which of the following statements concerning property is (are) correct?

1. A mortgage on real estate is real property.

2. A tree growing on land is real property.

A. Both 1 and 2
B. Neither 1 nor 2

C. 1 only

D. 2 only

Answer: D

QUESTION NO: 370

Nontax benefits of lifetime gifts include all the following EXCEPT to

A. reduce probate and administrative costs

B. provide for the education of the donee

C. avoid claims of creditors evidenced by liens

D. obtain privacy that is not possible to obtain through testamentary transfers

Answer: C

QUESTION NO: 371

A father died leaving his property equally to his wealthy son and his poor daughter. The son wishes to
disclaim his share of the inheritance so that it will pass to his sister without his incurring any gift tax
liability. In this situation, all the following acts on the part of the son are required EXCEPT:

A. His refusal to accept the inheritance must be received by the executor of his father's estate within 9
months of his father's death.

B. His refusal to accept the inheritance must direct specifically that his sister is to receive it instead.

C. His refusal to accept the inheritance must be in writing.


D. He must not have received any part of his inheritance or any income from it prior to his refusal to
accept it.

Answer: B

QUESTION NO: 372

A widower dies leaving a net probate estate of $300,000. At the time of his death, his descendants are
as follows:

A son, Joe, who has no children;

A deceased daughter, Mary, whose two children, Irene and Sally, survive; and

A daughter, Anne, who has one child, Harry

Assuming that the widower's will provides for the distribution of his assets in equal shares to his
children, per stirpes, which of the following correctly states the amounts each descendant will receive?

A. $100,000 to Joe, $50,000 to Irene, $50,000 to Sally, $50,000 to Anne, and $50,000 to Harry

B. $60,000 to Joe, $60,000 to Irene, $60,000 to Sally, $60,000 to Anne, and $60,000 to Harry

C. $75,000 to Joe, $75,000 to Irene, $75,000 to Sally, and $75,000 to Anne

D. $100,000 to Joe, $50,000 to Irene, $50,000 to Sally, and $100,000 to Anne

Answer: D

QUESTION NO: 373


Generally all the following statements concerning life insurance arrangements for partnership buy-sell
agreements are correct EXCEPT:

A. With an entity-purchase agreement the partnership strives to maintain face amounts of coverage
equal to its obligations under the agreement.

B. With a cross-purchase agreement each partner purchases life insurance on the life of each of the
other partners.

C. With an entity-purchase agreement the formula used for determining the number of life insurance
policies needed is N (number of partners) + 1.

D. With a cross-purchase agreement the individual partners are the applicants, owners, beneficiaries
and premium-payers of the policies.

Answer: C

QUESTION NO: 374

The federal gift tax is

A. levied directly on the gift

B. a flat tax based on gifts made in any taxable year

C. a tax on the right of the donee to receive the property

D. a tax on the right of the donor to make the gift

Answer: D

QUESTION NO: 375


A woman is the income beneficiary of an irrevocable trust. All the following powers held by her will
cause all the assets in the trust to be includible in her gross estate for federal estate tax purposes
EXCEPT

A. the testamentary power to direct the trustee to pay trust assets to her estate

B. the testamentary special or limited power to direct the trustee to distribute trust assets to her
children

C. the testamentary power to direct the trustee to use trust assets to pay her estate taxes

D. the power to direct the trustee to distribute trust corpus to her

Answer: B

QUESTION NO: 376

If a grantor establishes an irrevocable trust, the income of the trust will be taxed to the grantor if it is
used to pay premiums for life insurance on the life of

A. a child of the grantor

B. the spouse of the grantor

C. a grandchild of the grantor

D. the father of the grantor

Answer: B

QUESTION NO: 377

All the following statements concerning lifetime gifts are correct EXCEPT:
A. The amount of gift tax paid within 3 years of death is included in the gross estate.

B. If a wealthy widower lives more than 3 years after making a taxable gift to his sister, the value of the
gift has no effect on his federal estate tax liability.

C. A substantial amount of property may be given away over a period of time without the imposition of
the federal gift tax because of the annual exclusion.

D. Gifts of life insurance within 3 years of death are included in the donor-insured's gross estate.

Answer: B

QUESTION NO: 378

Which of the following statements concerning property ownership by a married couple residing in a
community-property state is correct?

A. All property owned by the couple is community property.

B. Property inherited by one spouse during a marriage becomes community property.

C. Community property loses its identity when a couple moves from a community-property state to a
common-law state.

D. Income earned by one spouse becomes community property.

Answer: D

QUESTION NO: 379

A father plans to create a trust for the benefit of his 22-year-old son and wishes to take advantage of the
gift tax annual exclusion. He has named a bank as trustee. Which of the following trust provisions would
cause the gifts to be ineligible to qualify for the gift tax annual exclusion?
1. The trust income is to be paid to the son or accumulated at the discretion of the trustee.

2. The income is to be accumulated until the son reaches age 32 when all accumulated income and
principal are to be distributed to him.

A. Both 1 and 2

B. Neither 1 nor 2

C. 1 only

D. 2 only

Answer: A

QUESTION NO: 380

A man established and funded an irrevocable trust and named a bank as trustee. All income from the
trust is to be paid to his four grandchildren. Which of the following powers retained by the grantor of
the trust will cause all or a portion of the trust assets to be includible in his gross estate for federal
estate tax purposes?

1. The power to add principal to the trust

2. The power to vary the amounts of trust income paid to each grandchild

A. Neither 1 nor 2

B. Both 1 and 2

C. 1 only
D. 2 only

Answer: D

QUESTION NO: 381

All the following statements concerning the generation-skipping transfer tax (GSTT) are correct EXCEPT:

A. All donors have a cumulative $1.5 million exemption against generation-skipping transfers.

B. The tax is imposed according to a graduated rate schedule similar to the federal estate and gift tax
rates.

C. The tax may be imposed on direct gifts to grandchildren.

D. The tax may be imposed on gifts in trust to grandchildren.

Answer: B

QUESTION NO: 382

A mother died leaving her property equally to her wealthy daughter and her poor son. The daughter
wishes to disclaim her share of the inheritance so that it will pass to her brother without her incurring
any gift tax liability. Assume that the disclaimer is written, is timely, and is irrevocable. Which of the
following statements concerning an additional requirement of a qualified disclaimer is (are) correct?

1. The daughter refusal must direct specifically that her brother is to receive her inheritance.

2. The daughter must not have received any part of her inheritance or any income from it prior to her
refusal to accept it.
A. 1 only

B. Neither 1 nor 2

C. Both 1 and 2

D. 2 only

Answer: D

QUESTION NO: 383

Which of the following members of the estate planning team is responsible for assuring that the client's
intentions are expressed in documents that will carry out the final plan?

A. The lawyer

B. The investment counselor

C. The trust officer

D. The life underwriter

Answer: A

QUESTION NO: 384

A number of states have passed statutes governing "living wills." Which of the following statements
concerning a living will is (are) correct?

1. A living will is an alternative to an inter vivos trust under certain circumstances.


2. A living will is an oral will made by the testator during a final illness when it is impossible to write one.

A. Both 1 and 2

B. 1 only

C. Neither 1 nor 2

D. 2 only

Answer: C

QUESTION NO: 385

Which of the following statements concerning filing the federal estate tax return is correct?

A. For persons dying under current law, an estate tax return must be filed for all U.S. citizen decedents.

B. A one-year extension for filing the estate tax return is granted when the estate contains a closely held
business interest.

C. An automatic two-year extension for filing the estate tax return is granted when the decedent dies
overseas.

D. The estate tax return must be filed within 9 months of death unless an extension is granted by the
IRS.

Answer: D

QUESTION NO: 386

Which of the following statements concerning the estate tax marital deduction is correct?
A. The marital deduction available to a decedent in a community-property state is equal to the total
amount of community property.

B. The marital deduction available to a decedent in a common-law state is equal to the net amount of
qualifying property passing to the surviving spouse.

C. The marital deduction available to a decedent in a common-law state is limited to a maximum of $1


million.

D. The marital deduction available to a decedent in a common-law state is equal to one half the adjusted
gross estate.

Answer: B

QUESTION NO: 387

On January 1, 2004 a father gave his daughter a $200,000 straight (ordinary) life insurance policy on his
life. Premiums are paid annually. The pertinent facts about the policy are:

 Date of issue: July 1, 1992

 Premium paid on July 1, 2003 $3,200

 Terminal reserve on July 1, 2003 20,000

 Terminal reserve on July 1, 2004 24,000

What is the value of the policy for federal gift tax purposes?
A. $200,000

B. $ 23,200

C. $ 23,600

D. $ 21,600

Answer: C

QUESTION NO: 388

A man recently died with only probate assets. Under the terms of his will, he left his entire probate
estate outright to his wife. The following are relevant facts about the estate:

 Gross estate $2,000,000


 Estate administration expenses 50,000
 Debts of decedent 200,000
 Allowable funeral expenses 5,000

The amount of the allowable marital deduction is

A. $1,745,000

B. $1,750,000

C. $1,795,000

D. $1,800,000

Answer: A
QUESTION NO: 389

Mr. Conrad died early this year. Under the terms of his will, he left all his real estate and tangible
personal property to his son. All the remainder of his probate estate was left to his wife, Mrs. Conrad.
The following is a list of Mr. Conrad's probate assets and their fair market values at the time of his
death:

 Commercial real estate $150,000


 Furniture and fixtures 50,000
 Listed common stock 100,000
 Other securities 200,000

In addition, Mr. Conrad also owned a $300,000 life insurance policy on his life with Mrs. Conrad
designated as beneficiary. Based on this information, what is the amount of property in Mr. Conrad's
estate qualifying for the federal estate tax marital deduction?

A. $800,000

B. $600,000

C. $300,000

D. $400,000

Answer: B

QUESTION NO: 390

A man died in February of this year. Last year, when he learned that he had terminal illness, he
immediately made the following gifts and filed the required gift tax return:

Fair Market Value

Gift of listed stock to


 a qualified charity $200,000
 Gift of listed bonds to his wife 300,000
 Gift of a boat to his son 10,000
 Gift of a sports car to his daughter 10,000

What amount must be brought back to the man's estate as an adjusted taxable gift in the calculation of
his federal estate taxes?

A. $520,000

B. $290,000

C. $200,000

D. 0

Answer: D

QUESTION NO: 391

All the following statements concerning antenuptial agreements are correct EXCEPT:

A. They can often deal with a surviving spouse's right to an intestate share of a deceased spouse's
estate.

B. They are frequently used prior to second marriages.

C. They can protect the interests of children of former marriages.

D. Income tax problems are avoided when property is transferred subject to such an agreement.

Answer: D
QUESTION NO: 392

Which of the following statements concerning charitable remainder annuity trusts is correct?

A. The interest paid to the charity must be paid at least quarterly.

B. It provides a fixed annuity income interest to a qualified charity.

C. The term of charitable remainder annuity arrangements is limited to 10 years.

D. The donor receives an estate tax charitable deduction for the value of the remainder interest.

Answer: D

QUESTION NO: 393

A man established and funded an irrevocable trust and named a bank as trustee. All income from the
trust is to be paid to his four grandchildren. Which of the following powers retained by the grantor of
the trust will cause all or a portion of the trust assets to be includible in his gross estate for federal
estate tax purposes?

1. The power to add principal to the trust

2. The power to vary the amounts of trust income paid to each grandchild

A. 1 only

B. 2 only

C. Neither 1 nor 2

D. Both 1 and 2
Answer: B

QUESTION NO: 394

A man is planning to establish and fund an irrevocable trust for the benefit of his two sons, ages 19 and
22, and plans to give the trustee power to sprinkle trust income. From the standpoint of providing
federal income, gift, and estate tax savings, which of the following would be a suitable trustee?

1. The grantor of the trust

2. The grantor's 22-year-old son

A. 2 only

B. Both 1 and 2

C. Neither 1 nor 2

D. 1 only

Answer: C

QUESTION NO: 395

All the following statements concerning transfers at death under a will are correct EXCEPT:

A. Specific bequests of a decedent's property are satisfied prior to distribution of the decedent's
residuary estate.

B. The most appropriate way to sever a joint tenancy with right of survivorship is for the joint tenant-
decedent to make a specific bequest of the property under a will.
C. It is common for a will to contain a clause that exempts the executor from posting bond.

D. If during lifetime a decedent disposed of property that was the subject of a specific bequest,
ademption occurs.

Answer: B

QUESTION NO: 396

The personal representative of a decedent has the duty to file which of the following income tax
returns?

1. The decedent's final income tax return

2. The estate's income tax return

A. Both 1 and 2

B. Neither 1 nor 2

C. 2 only

D. 1 only

Answer: A

QUESTION NO: 397

The decedent, D, died this year. The facts concerning D estate are:
 Gross estate $3,400,000
 Marital deduction 0
 Charitable deduction 600,000
 Funeral & administration expenses 80,000
 Gifts made after 1976 170,000
 State death taxes payable 192,000

What is D taxable estate?

A. $2,720,000

B. $2,358,000

C. $2,138,000

D. $2,528,000

Answer: D

QUESTION NO: 398

All the following statements concerning a federal estate tax deduction for a bequest or gift to a qualified
charity are correct EXCEPT:

A. A life insurance policy that was assigned to a charity as a gift less than 3 years prior to the insured's
death qualifies for a charitable deduction.

B. The amount of a charitable deduction may not exceed 50 percent of a decedent's adjusted gross
estate.

C. The amount of a charitable deduction is reduced by any taxes and administrative expenses chargeable
against the bequest.

D. An estate may deduct the value of the remainder interest in a charitable remainder trust.
Answer: B

QUESTION NO: 399

Which of the following statements concerning a simple trust is correct?

A. Income and principal may be distributed to a qualified charity.

B. Income is accumulated at the discretion of the trustee.

C. It receives a special tax deduction for income distributed to its beneficiaries.

D. It limits the number of permissible beneficiaries.

Answer: C

QUESTION NO: 400

Which of the following actions on the part of a trustee is (are) a breach of his duties?

1. Placing substantial amounts of cash from the sale of securities in a noninterest bearing checking
account for a period of years

2. Investing all trust assets in securities that favor income beneficiaries to the detriment of
remainderpersons

A. 1 only

B. Neither 1 nor 2

C. Both 1 and 2
D. 2 only

Answer: C

You might also like